From 565a46a1bdc5e2d41c84a47adeb95363d36f4129 Mon Sep 17 00:00:00 2001 From: Ulli Kehrle Date: Sat, 30 Dec 2017 15:01:41 +0100 Subject: updated everything --- ch01-lineare-struktur.tex | 155 ++++++------- ch02-topologie.tex | 187 ++++++++++++---- ch03-topologisch-lineare-raeume.tex | 429 +++++++++++++++++++++--------------- ch05-hahn-banach.tex | 17 +- ch06-schwache-topologien.tex | 30 +-- common.tex | 39 +++- funkana.tex | 10 +- motivation.tex | 4 +- ref.bib | 9 + skript.cls | 88 +++++--- 10 files changed, 620 insertions(+), 348 deletions(-) diff --git a/ch01-lineare-struktur.tex b/ch01-lineare-struktur.tex index bd2f259..3944b47 100644 --- a/ch01-lineare-struktur.tex +++ b/ch01-lineare-struktur.tex @@ -1,9 +1,10 @@ \chapter{Die lineare Struktur} \label{cha:die-lineare-struktur} \index{Struktur!lineare} +Alle in diesem Kapitel vorgestellten Resultate gelten für beliebige Körper. +Wir werden uns aber im weiteren Verlauf quasi ausschließlich mit den aus der Analysis bekannten Körper der reellen Zahlen $ℝ$ und der komplexen Zahlen $ℂ$ beschäftigen. \section{Der lineare Raum} \label{sec:der-lineare-raum} -Sei im Folgenden stets $\K = ℝ$ oder $\K = ℂ$. Zunächst die \begin{definition}[Vektorraum, linearer Raum] \label{defi:vektorraum-1.1.1} \index{Raum!linearer} @@ -13,55 +14,52 @@ Sei im Folgenden stets $\K = ℝ$ oder $\K = ℂ$. Zunächst die \cdot : \K × X → X \] heißt $\K$-\emph{Vektorraum} oder \emph{linearer Raum}, falls für alle $\alpha , β ∈ \K$ und $x, y ∈ X$ gilt: - \begin{enumerate}[label=(V\arabic*)] + \begin{wenumerate}[label=(V\arabic*)] \item $\alpha x+y) = \alpha x + βy$ \item $(\alpha +β)x = \alpha x + βx$ \item $(\alpha β)x = \alpha (βx)$ \item $1 \cdot x = x$ - \end{enumerate} + \end{wenumerate} \end{definition} -\begin{bemerkung-nn} - Je nachdem, ob $\K = ℂ$ oder $\K = ℝ$ gilt, heißt $X$ ein \emph{komplexer} oder ein \emph{reeller} Vektorraum. -\end{bemerkung-nn} -\begin{bemerkung-nn} - \index{Raum!linearer Teil-} - Eine nichtleere Teilmenge $Y ⊂ X$ ist bereits dann ein linearer Raum, falls aus $\alpha , β ∈ \K$, $x, y ∈ Y$ bereits $\alpha x + βy ∈ Y$ folgt, also $Y$ abgeschlossen unter den Vektorraumoperationen ist. - $Y$ heißt dann \emph{linearer Teilraum} oder auch \emph{linearer Unterraum}. -\end{bemerkung-nn} -\begin{bemerkung-nn} - \index{Aufspann} - Zu jeder Teilmenge $M ⊂ X$ bildet die Menge aller Linearkombinationen von je endlich vieler Elemente einen linearen Teilraum von $X$. - Dieser heißt die \emph{lineare Hülle} von $M$ oder der \emph{Aufspann} von $M$ - \[ - \lspan M = \left\{ x ∈ X: ∃ l ∈ ℕ, \alpha _1,…,\alpha _l ∈ \K, m_1,…,m_l ∈ M \text{ mit } \sum_{i=1}^l \alpha _i m_i = x \right\}. - \] -\end{bemerkung-nn} -\begin{bemerkung-nn} - \index{Basis!Hamel-} - $M = \{x_\lambda \}_{\lambda ∈ \Lambda } ⊂ X$ heißt \emph{Basis} oder \emph{Hamel-Basis} von $X$, falls $M$ \emph{linear unabhängig}, das heißt, - $0 ∈ X$ lässt sich nur auf triviale Art und Weise als Linearkombination endlich vieler der $x_\lambda $ schreiben, und $\lspan M = X$ ist. -\end{bemerkung-nn} -\begin{bemerkung-nn} - \index{Dimension} - Besitzt $X$ eine Basis von $n < \infty $ Elementen, dann heißt $n$ die \emph{Dimension} von $X$ und wir schreiben $\dim X = n$. - Andernfalls heißt $X$ \emph{unendlich-dimensional} ($\dim X = \infty $). -\end{bemerkung-nn} -\begin{bemerkung-nn} - \index{Summe} - Seien $X_1, X_2 ⊂ X$ lineare Teilräume. Dann ist - \[ - X_1 + X_2 \coloneq \left\{ \alpha x_1 + βx_2: \alpha , β ∈ \K, x_1 ∈ X_1, x_2 ∈ X_2 \right\} - \] - ebenfalls ein linearer Teilraum. - Falls $X_1 ∩ X_2 = \{ 0\}$, schreiben wir $X_1 \oplus X_2$ und nennen die Summe \emph{direkt}. -\end{bemerkung-nn} -\begin{bemerkung-nn} - \index{Raum!Quotienten-} - Sei $Y$ ein linearer Teilraum von $X$. Definiere die Äquivalenzrelation $\sim$ auf $X$ durch - $x \sim y \Leftrightarrow x - y ∈ Y$. - Dann wird die Menge der Äquivalenzklassen mit vertreterweiser Addition und Multiplikation auch ein $\K$-Vektorraum. - Wir schreiben für diesen Vektorraum $X/Y$. -\end{bemerkung-nn} +\begin{definition-nn} + \begin{enumerate} + \item + Je nachdem, ob $\K = ℂ$ oder $\K = ℝ$ gilt, heißt $X$ ein \emph{komplexer} oder ein \emph{reeller} Vektorraum. + \item + \index{Raum!linearer Teil-} + Eine nichtleere Teilmenge $Y ⊂ X$ ist bereits dann ein linearer Raum, falls aus $\alpha , β ∈ \K$, $x, y ∈ Y$ bereits $\alpha x + βy ∈ Y$ folgt, also $Y$ abgeschlossen unter den Vektorraumoperationen ist. + $Y$ heißt dann \emph{linearer Teilraum} oder auch \emph{linearer Unterraum}. + \item + \index{Aufspann} + Zu jeder Teilmenge $M ⊂ X$ bildet die Menge aller Linearkombinationen von je endlich vieler Elemente einen linearen Teilraum von $X$. + Dieser heißt die \emph{lineare Hülle} von $M$ oder der \emph{Aufspann} von $M$ + \[ + \lspan M = \Big\{ x ∈ X: ∃ l ∈ ℕ, \alpha _1,…,\alpha _l ∈ \K, m_1,…,m_l ∈ M \text{ mit } \sum_{i=1}^l \alpha _i m_i = x \Big\}. + \] + \item + \index{Basis!Hamel-} + $M = \{x_\lambda \}_{\lambda ∈ \Lambda } ⊂ X$ heißt \emph{Basis} oder \emph{Hamel-Basis} von $X$, falls $M$ \emph{linear unabhängig}, das heißt, + $0 ∈ X$ lässt sich nur auf triviale Art und Weise als Linearkombination endlich vieler der $x_\lambda $ schreiben, und $\lspan M = X$ ist. + \item + \index{Dimension} + Besitzt $X$ eine Basis von $n < \infty $ Elementen, dann heißt $n$ die \emph{Dimension} von $X$ und wir schreiben $\dim X = n$. + Andernfalls heißt $X$ \emph{unendlich-dimensional} ($\dim X = \infty $). + \item + \index{Summe} + Seien $X_1, X_2 ⊂ X$ lineare Teilräume. Dann ist + \[ + X_1 + X_2 \coloneq \left\{ \alpha x_1 + βx_2: \alpha , β ∈ \K, x_1 ∈ X_1, x_2 ∈ X_2 \right\} + \] + ebenfalls ein linearer Teilraum. + Falls $X_1 ∩ X_2 = \{ 0\}$, schreiben wir $X_1 \oplus X_2$ und nennen die Summe \emph{direkt}. + \item + \index{Raum!Quotienten-} + Sei $Y$ ein linearer Teilraum von $X$. Definiere die Äquivalenzrelation $\sim$ auf $X$ durch + $x \sim y \Leftrightarrow x - y ∈ Y$. + Dann wird die Menge der Äquivalenzklassen mit vertreterweiser Addition und Multiplikation auch ein $\K$-Vektorraum. + Wir schreiben für diesen Vektorraum $X/Y$. + \end{enumerate} +\end{definition-nn} \begin{satz} \label{satz:vr-besitzt-basis-1.1.2} Jeder lineare Raum besitzt eine (Hamel-)Basis. @@ -87,12 +85,22 @@ Sei im Folgenden stets $\K = ℝ$ oder $\K = ℂ$. Zunächst die \section{Beispiele} \label{sec:beispiele} +In diesem Abschnitt geben geben wir nun einige Beispiele zu linearen Räumen über den Körpern $ℝ$ und $ℂ$ an. +Wir werden uns mit diesen Räumen noch weiter beschäftigen, zunächst betrachten wir aber nur die lineare Struktur auf ihnen. +Zunächst die (bis auf isomorphie eindeutig bestimmten) endlich"=dimensionalen Räume: \index{$ℝ^n$} -\begin{beispiel} +\index{$ℂ^n$} +\begin{beispiel}[$ℝ^n$, $ℂ^n$] Der $ℝ^n$ ist ein linearer Raum über dem Körper $ℝ$. Der $ℂ^n$ ist sowohl ein $ℂ$- als auch ein $ℝ$-Vektorraum. + Dabei ist $\dim_ℝ ℝ^n = n$ und $\dim_ℂ ℂ^n = n$, aber $\dim_ℝ ℂ^n = 2n$. + Insbesondere ist $C$ auch ein zwei"=dimensionaler reeller Vektorraum. \end{beispiel} -\begin{beispiel} +In der klassischen Analysis haben wir uns bereits ausgiebigst mit diesen Räumen befasst. +Die Funktionalanlaysis versucht nun, einige der Konzepte, die wir von diesen Räumen kennen, auf die nachfolgenden unendlich"=dimensionalen Räume zu übertragen: + +\begin{beispiel}[{$C[a,b]$}] + \index{$C[a,b]$} Sei $[a,b] ⊂ ℝ$, $a < b$. Dann ist \[ C[a,b] = \{x: [a,b] → \K, x \text { ist stetig}\} @@ -106,32 +114,34 @@ Sei im Folgenden stets $\K = ℝ$ oder $\K = ℂ$. Zunächst die \index{$\ell^p$} \index{Folge!$p$-summierbar} \index{Raum!Folgen-} - Es ist + Sei $0 < p < ∞$. Wir betrachten die Menge $\ell^p$ aller $p$-Summierbaren Folgen in $\K$ \[ - \ell^p = \{ x = (ξ_n)_{n ∈ ℕ} : ξ_n ∈ \K, \sum_{n=1}^∞ |ξ_n|^p < ∞ \} + \ell^p = \Big\{ x = (ξ_n)_{n ∈ ℕ} : ξ_n ∈ \K, \sum_{n=1}^∞ |ξ_n|^p < ∞ \Big\}. \] - für $0 < p < ∞$ ein linearer Raum. - Die Menge der Einheitsvektoren $\{e_i\}_{i ∈ ℕ}$ mit $e_i = (0,…,0,1,0,…)$ ist eine unendliche linear unabhängige Teilmenge, aber ebenfalls keine Basis. + Sie wird mit der komponentenweisen Addition und Skalarmultiplikation ein linearer Raum. + Dabei ist die Menge der Einheitsvektoren $\{e_i\}_{i ∈ ℕ}$ mit $e_i = (0,…,0,1,0,…)$ ist eine unendliche linear unabhängige Teilmenge, aber ebenfalls keine Basis. Genauso ist \[ - \ell^∞ = \left\{ x = (ξ_n)_{n ∈ ℕ} : ξ_n ∈ \K, \sup_{n=1} |ξ_n| < ∞ \right\} + \ell^∞ = \Big\{ x = (ξ_n)_{n ∈ ℕ} : ξ_n ∈ \K, \sup_{n=1} |ξ_n| < ∞ \Big\} \] - ein überabzählbar"=dimensionaler linearer Raum mit den Unterräumen + ein überabzählbar"=dimensionaler linearer Raum mit den unendlich"=dimensionalen linearen Unterräumen \[ - c = \left\{ (ξ_n)_{n ∈ ℕ} ∈ \ell^∞: \lim_{n → ∞} ξ_n \text{ existiert}\right\} + c = \Big\{ (ξ_n)_{n ∈ ℕ} ∈ \ell^∞: \lim_{n → ∞} ξ_n \text{ existiert}\Big\} \] und \[ - c_0 = \left\{ (ξ_n)_{n ∈ ℕ} ∈ \ell^∞: \lim_{n → ∞} ξ_n = 0 \right\}. + c_0 = \Big\{ (ξ_n)_{n ∈ ℕ} ∈ \ell^∞: \lim_{n → ∞} ξ_n = 0 \Big\}. \] \end{beispiel} -\begin{beispiel}[Lebesgue-integrierbare Funktionen] \index{$L^p$} +\begin{beispiel}[Lebesgue-integrierbare Funktionen] + \index{$L^p$} + \index{$\L^p$} \index{Funktion!Lebesgue-integrierbar} Sei $M ⊂ ℝ$ messbar und $0 < p < ∞$. Dann ist \[ - \L^p(M) = \left\{f : M → ℝ, f \text { messbar}, ∫_M |f|^p \dd μ < ∞ \right\} + \L^p(M) = \Big\{f : M → ℝ, f \text { messbar}, ∫_M |f|^p \dd μ < ∞ \Big\} \] ein unendlich"=dimensionaler linearer Raum. Offenkundig ist $\mathcal N \coloneq \{ f: M → ℝ,\; f = 0$ fast überall $\}$ ein Unterraum von $\L^p(M)$, also auch @@ -184,7 +194,7 @@ Sei im Folgenden stets $\K = ℝ$ oder $\K = ℂ$. Zunächst die \end{enumerate} \end{bemerkung} -\begin{beispiel-nn} +\begin{beispiel} $X = \{ x: [a,b] → ℝ, x, \dot x, \ddot x \text{ stetig},\; x(a) = \dot x(a) = 0\}$ ist ein linearer Raum. Sei $Y = C[a,b]$ und $A: X → Y$ gegeben durch \[ @@ -201,9 +211,9 @@ Sei im Folgenden stets $\K = ℝ$ oder $\K = ℂ$. Zunächst die Also ist $A$ bijektiv, das heißt, es gibt eine lineare Abbildung $A^{-1}: Y → X$. Diese Inverse ist in der Regel schlecht anzugeben. Einen einfacheren Spezialfall dazu wird in der Übung behandelt. -\end{beispiel-nn} +\end{beispiel} -\begin{beispiel-nn} +\begin{beispiel} Sei $X = Y = C[a,b]$, $A: X → X$ gegeben durch \[ (Ax)(t) \coloneq ∫_a^b k(s,t) x(s) ds, \quad t ∈ [a,b], @@ -217,9 +227,9 @@ Sei im Folgenden stets $\K = ℝ$ oder $\K = ℂ$. Zunächst die linear. Die Probleme $Ax = y$ (bei gegebenem $y ∈ Y$ und gesuchtem $x ∈ X$) oder $A_\lambda x = 0$ (gesucht ist $\lambda ∈ ℝ$ und eine nichttriviale Lösung $x ∈ X \setminus \{ 0\}$) heißen Integralgleichungen erster und zweiter Ordnung. -\end{beispiel-nn} +\end{beispiel} -\begin{beispiel-nn} +\begin{beispiel} Sei $X = C[a,b]$, $A : X → ℝ$ mit \[ Ax = x(t_0), @@ -230,28 +240,25 @@ Sei im Folgenden stets $\K = ℝ$ oder $\K = ℂ$. Zunächst die Ax = ∫_a^b x(t) dt \] Dann sind beide Abbildungen $A$ linear und nicht injektiv, aber surjektiv. -\end{beispiel-nn} +\end{beispiel} -\begin{beispiel-nn} +\begin{beispiel} Sei $X = \ell^2$, $A: X → X$. Für $x = (ξ_n)_{n ∈ ℕ}$ sei \[ Ax = (0,ξ_1, ξ_2, \dots) ∈ \ell^2. \] $A$ heißt (Rechts-)Shiftoperator und ist linear und injektiv, jedoch nicht surjektiv. Solche Abbildungen gibt es für $\dim X = \dim Y < \infty $ nicht. -\end{beispiel-nn} +\end{beispiel} \section{Duale Räume} \label{sec:duale-raume} -$A: X → \K$ sei ein lineares Funktional, $X$ ein linearer Raum. Wir verwenden ein neues Symbol (statt $A$) -\[ - x': X → \K \text{ linear}. -\] +Wir bezeichnen lineare Funktionale $X → \K$ (also stetige lineare Abbildungen $X → \K$) üblicherweise mit $x'$. Wir schreiben nun \[ - x'(x) \eqcolon \langle x, x' \rangle = \langle x, x' \rangle_{X × X^f} ∈ \K. + x'(x) \eqcolon \langle x, x' \rangle = \langle x, x' \rangle_{X × X^f} ∈ \K \] -Wir setzen +und setzen \[ X^f \coloneq \left\{ x': x' \text{ ist lineares Funktional auf } X \right\}. \] @@ -262,11 +269,11 @@ Der Raum $X^f$ wird auf natürlicher Weise zum linearen Raum mit \[ (\alpha x_1' + βx_2')(x) \coloneq \alpha x_1'(x) + βx_2'(x), \quad x ∈ X, x_1', x_2' ∈ X^f, \alpha , β ∈ \K. \] -So ist +Dann ist \[ \langle -,- \rangle_{X×X^f}: X × X^f → \K \] -bilinear. +eine Bilinearform. \begin{definition}[Algebraischer Dualraum, Algebraischer Bidualraum] \index{Raum!algebraischer Dual-} \index{Raum!algebraischer Bidual-} @@ -320,5 +327,5 @@ Um Allerdings von Stetigkeit reden zu können, müssen wir zunächst \emph{Topol %%% Local Variables: %%% mode: latex -%%% TeX-master: "funkana-ebook" +%%% TeX-master: "funkana" %%% End: diff --git a/ch02-topologie.tex b/ch02-topologie.tex index bf094ab..e91a20d 100644 --- a/ch02-topologie.tex +++ b/ch02-topologie.tex @@ -2,7 +2,13 @@ \label{cha:topologie} \section{Topologische Räume} \label{sec:topologische-raume} -\begin{definition}[Topologischer Raum, offene Mengen] + +\begin{definition-nn}[Potenzmenge] + \index{$\Pot X$} + \index{Potenzmenge} + Wir bezeichnen für eine Menge $X$ mit $\Pot X$ die Potenzmenge $\{ M: M ⊂ X \}$ von $X$. +\end{definition-nn} +\begin{definition}[Topologie, Topologischer Raum, offene Mengen] \index{Raum!topologischer} \index{Struktur!topologische} \index{offen} @@ -40,6 +46,7 @@ Der \emph{Sierpinski-Raum} ist die Menge $\{0,1\}$ versehen mit der Topologie $\{ ∅, \{0\}, \{0,1\}\}$. \end{enumerate} \end{beispiele-nn} +Wir führen zunächst einige wichte Begriffe ein: \begin{definition} \label{defi:top-grundbegriffe-2.1.2} Sei $M ⊂ X$. @@ -129,8 +136,8 @@ \end{definition} \begin{bemerkung-nn} Man überlegt sich leicht, dass der Grenzwert $x_{0}$ in der Regel nicht eindeutig ist. - Bsp: In $\T=\{X,\emptyset\}$ konvergiert jede Folge gegen jeden Punkt. - Ist $(X,\T)$ jedoch ein Hausdorff-Raum, so ist jeder Grenzwert eindeutig. + In der Klumpentopologie $\T=\{X,\emptyset\}$ konvergiert jede Folge gegen jeden Punkt. + Ist $(X,\T)$ jedoch ein Hausdorff"=Raum, so ist jeder Grenzwert eindeutig. \end{bemerkung-nn} \begin{beweis} Seien $x_{0} \neq x'_{0}$ Grenzwerte von $(x_{n})_{n \in \N} \subset X$. @@ -149,11 +156,12 @@ \end{definition} \begin{beispiel-nn} Wir betrachten $\R$ mit der natürlichen Topologie. - $(x_n)_{n ∈ ℕ}$ mit $x_n=(-1)^n$ hat zwei Häufungspunkte $\pm 1$. + Die Folge $(x_n)_{n ∈ ℕ}$ mit $x_n=(-1)^n$ hat zwei Häufungspunkte $1$ und $-1$. Die Menge aller Folgenglieder $M=\{x_{n}:n \in \N\}=\{-1,1\}$ hat als Menge jedoch keine Häufungspunkte. \end{beispiel-nn} \begin{bemerkung-nn} - Für die indiskrete Topologie ist jeder Punkt in $X$ Häufungspunkt jeder Folge. + Die Anzahl der Häufungsunkte, die eine Folge haben kann, ist unbeschränkt. + Ist $X$ eine beliebige Menge, die mit der Klumpentopologie ausgestattet ist, so ist jeder Punkt in $X$ Häufungspunkt (und Grenzwert) jeder Folge. \end{bemerkung-nn} \begin{definition}[Stetigkeit] \index{stetig} @@ -168,9 +176,12 @@ $f$ heißt \emph{stetig}, falls für alle $V \in \T_{Y}$ gilt, dass $f^{-1}(V) \in \T_{X}$. \end{enumerate} \end{definition} -\begin{bemerkung-nn} - $f$ ist genau dann stetig, wenn $f$ in jedem Punkt stetig ist. -\end{bemerkung-nn} +\begin{lemma-nn} + Eine Abbildung $f: X → Y$ zwischen topologischen Räumen $X$, $Y$ ist genau dann stetig, wenn $f$ in jedem Punkt stetig ist. +\end{lemma-nn} +\begin{noproof} + ~ +\end{noproof} \begin{definition}[Homöomorphismus] \index{Homöomorphismus} \index{isomorph!topologisch} @@ -195,7 +206,7 @@ existiert. \begin{beispiel-nn} Für die natürliche Topologie auf $\R^n$ ist eine Basis der Topologie gegeben durch ${B_{\eps}(x): x \in X, \eps > 0}$ - mit den offenen Kugeln $B_{\eps}(x)={y \in R^n : \norm{x-y}<\eps}$. + mit den offenen Kugeln $B_{\eps}(x)={y \in R^n : \snorm{x-y}<\eps}$. Sei $x \in \R^n$ fest. Dann ist ${B_{1/n}(x):n \in \N}$ eine abzählbare Umgebungsbasis von x \end{beispiel-nn} @@ -203,12 +214,12 @@ existiert. \index{Topologie!Relativ-} \index{Topologie!Spur-} \label{defi:relativtop-2.1.10} - $M \subset \T$ eines topologischen Raumes $(X,\T)$ lässt sich in natürlicher Weise + Eine Teilmenge $M \subset \T$ eines topologischen Raumes $(X,\T)$ lässt sich in natürlicher Weise zu einem topologischen Raum machen, nämlich mit $\T' \coloneq \{M \cap V : V \in \T\}$. \end{definition} \begin{bemerkung-nn} - $M = M \cap X \in \T'$ da $X \in \T$, d.h. $M$ ist offen in der Spurtopologie. - Achtung: $M$ muss nicht offen in $X$ sein. + Es gilt $M = M \cap X \in \T'$, denn es ist $X \in \T$, das heißt $M$ ist offen in der Spurtopologie (denn $M$ muss ja in jeder Topologie auf $M$ offen sein) + Aber $M$ muss hingegen nicht notwendigerweise offen in $X$ sein. \end{bemerkung-nn} \begin{definition} \index{Topologie!feiner} @@ -224,17 +235,30 @@ existiert. Die feinere Topologie $\T_{1}$ enthält mehr offene Mengen, und damit zu jedem Grenzwert $x_{0}$ weniger konvergte Folgen. - Man zeigt leicht: - $\T_{1}$ ist feiner als $\T_{2}$ $\Longleftrightarrow$ - Für alle $x \in X$ gilt: Seien $B_{1} \subset T_{1},B_{2} \subset T_{2}$ Umgebungsbasen von $x$, - dann gilt für alle $U \in B_{1}$, dass ein $V \in B_{2}$ existiert mit $V \subset U$. \end{bemerkung-nn} +\begin{lemma-nn} + Sei $X$ eine Menge, $\T_1$ und $\T_2$ Topologien auf $X$, für jedes $x ∈ X$ $B^x_i$ eine Umgebungsbasis von $x$ in $\T_i$, $i=1,2$. + Dann sind äquivalent: + \begin{enumerate} + \item + $\T_{1}$ ist feiner als $\T_{2}$ + \item + Für alle $x ∈ X$ und jedes $U ∈ \U_x^{\T_1}$ gibt es ein $V ∈ \U_x^{\T_2}$ mit $V ⊂ U$. + \item + Für alle $x \in X$ gibt es für jedes $U \in B_{1}$ ein $V \in B_{2}$ mit $V \subset U$. + \end{enumerate} +\end{lemma-nn} +\begin{noproof} + ~ +\end{noproof} \begin{beispiel-nn} - Folgende Topolgien auf $\R^n$ sind gleich. + Folgende Topolgien $\T_1$ und $\T_2$ auf $\R^n$ sind gleich: $\T_{1}$ sei die Topologie, die durch die Kugeln $B_{\eps}(x)=\{y \in R^n : \norm{x-y}<\eps\}$ erzeugt wird. $\T_{2}$ sei die Topologie, die durch die Quader $B_{\eps}(x)=\{y \in R^n : \max_{1 \leq i \leq n} |y_{i}-x_{i}|<\eps\}$ erzeugt wird. + Tatsächlich sind alle Topologien auf $ℝ^n$, die durch eine Norm induziert + werden identisch, denn alle Normen auf dem $ℝ^n$ sind uniform äquivalent. \end{beispiel-nn} \begin{definition}[Produkttopologie] \index{Topologie!Produkt-} @@ -244,8 +268,29 @@ existiert. \{U_{X} \times U_{Y} : U_{X} \in \T_{X}, U_{Y} \in \T_{Y} \} \subset \Pot{X \times Y} \] eine Basis der Topologie $\T_{X \times Y}$ im kartesischen Produkt $X \times Y$. - Bemerkung: Es genügt auch wenn $U_{X},U_{Y}$ über Basen von $\T_{X},\T_{Y}$ genommen werden. \end{definition} +\begin{bemerkung-nn} + In dieser Definition würde es auch genügen, wenn $U_{X},U_{Y}$ über Basen von $\T_{X},\T_{Y}$ genommen werden. +\end{bemerkung-nn} +\begin{bemerkung-nn} + Allgemeiner kann man auch das Produkt beliebig vieler topologischen Räume auf natürliche Art und Weise mit einer Topologie ausstatten: +\end{bemerkung-nn} +\begin{definition-nn}[Produkttopologie] + Sei $(X_i, \T_i)_{i ∈I}$ eine nichtleere Familie topologischer Räume. + Das kartesische Produkt $\prod_{i ∈ I} X_i$ ist die Menge + \[ + X = \prod_{i ∈ I} X_i = \{ f: I → \bigcup_{i ∈ I} X_i: ∀i ∈ I : f(i) ∈ X_i\}. + \] + Man schreibt die Elemente des kartesischen Produktes als Familien $(x_i)_{i ∈ I}$ mit $x_i ∈ X_i$ für alle $i ∈ I$. + Die \emph{kanonischen Projektionen} $\operatorname{pr}_i: X → X_i$ sind dann gerade die Abbildungen $\operatorname{pr}_i((x_j)_{j∈ I}) = x_i$. + Die zu $(\T_i)_{i ∈ I}$ gehörende \emph{Produkttopologie} ist dann die gröbste Topologie auf $X$, die die Abbildungen $\operatorname{pr}_i : X → X_i$ stetig macht. + Eine Basis der Produkttopologie ist gegeben durch die Zylindermengen + \[ + \mathcal B = \left\{ \prod_{i ∈ I} U_i: U_i ∈ \T_i \text{ und fast alle $U_i = X_i$ } \right\}. + \] + Das kartesische Produkt von endlich viele topologischen Räumen, etwa $X, Y, Z$ schreibt man als $X × Y × Z$. + Es gilt $X × (Y × Z) \simeq X × Y × Z \simeq (X × Y) × Z$. +\end{definition-nn} \section{Metrische Räume} \index{Raum!metrischer} \label{sec:metrische-raume} @@ -255,7 +300,7 @@ existiert. \label{defi:metrik-2.2.1} Sei $X$ eine Menge. $d: X × X → \R$ heißt \emph{Pseudometrik}, wenn $d$ den folgenden Axiomen genügt: - \begin{enumerate}[series=metrik,label=(M\arabic*)] + \begin{wenumerate}[series=metrik,label=(M\arabic*)] \item Für alle $x, y ∈ X$ gilt $d(x,y) \ge 0$ und $d(x,x) = 0$. \item @@ -264,12 +309,12 @@ existiert. \index{Dreiecksungleichung} \emph{Dreiecksungleichung:} Für alle $x, y, z ∈ X$ gilt $d(x,y) \le d(x,y) + d(z,y)$. - \end{enumerate} + \end{wenumerate} $d$ heißt \emph{Metrik}, falls es zusätzlich - \begin{enumerate}[resume=metrik,label=(M\arabic*)] + \begin{wenumerate}[resume=metrik,label=(M\arabic*)] \item $d(x,y) = 0 \implies x = y$ - \end{enumerate} + \end{wenumerate} \index{Kugel!offene} erfüllt. $(X,d)$ heißt dann (pseudo-)metrischer Raum. Zu $x ∈ X$ und $r > 0$ definieren wir die \emph{offene Kugel um $x$ mit Radius $r$} als @@ -326,10 +371,11 @@ existiert. Dann sind $B_{δ/2}(x)$ und $B_{δ/2}(y)$ disjunkte Umgebungen von $x$ bzw $y$: Sei $z ∈ B_{δ/2}(x)$. Dann ist \[ - d(z,y) \ge d(y,x) - d(x,z) > δ - \frac {δ} 2 = \frac {δ} 2. + d(z,y) \ge d(y,x) - d(x,z) > δ - δ/2 = δ/2. \] \end{proof} \begin{lemma-nn}[Eigenschaften metrischer Räume] + \label{lemma:eigenschaften-metrischer-raeume} Sei $(X,d)$ ein metrischer Raum. \begin{enumerate} \item Jeder Punkt $x ∈ X$ besitzt eine abzählbare Umgebungsbasis @@ -345,6 +391,20 @@ existiert. Es ist $x_0 ∈ M$ genau dann ein innerer Punkt von $M ⊂ X$, wenn ein $\epsilon > 0$ existiert mit $B_\epsilon (x_0) ⊂ M$. \item $M$ ist nirgends dicht in $X$ genau dann, wenn es zu jeder Kugel $B_\epsilon (x_0)$ mit $x_0 ∈ X, \epsilon > 0$ eine Kugel $B_\delta (x_1) ⊂ B_\epsilon (x_0)$ mit $B_\delta(x_1) ∩ M = \emptyset$ gibt. + \item + Seien $(X,d_X)$ und $(Y,d_Y)$ metrische Räume, $f: X → Y$ eine Abbildung, $x ∈ X$. + Dann sind äquivalent: + \begin{enumerate} + \item + $f$ ist steitg in $x$. + \item + $f$ ist folgenstetig in $x$. + \item + Für jedes $ε > 0$ existiert ein $δ > 0$, so dass für alle $y ∈ X$ gilt: + \[ + d(x,y) < δ \implies d(f(x), f(y)) < ε. + \] + \end{enumerate} \item Seien $(X,d_X)$ und $(Y,d_Y)$ metrische Räume. Dann ist auch $(X×Y,d_{X×Y})$ ein metrischer Raum vermöge der Metrik @@ -375,13 +435,15 @@ Der Beweis wird aufgrund seiner Trivialität den Lesern zur Übung überlassen, Nun ein paar Charakterisierungen von kompakten Mengen in metrischen Räumen. \begin{satz} \index{kompakt} + \index{kompakt!in metrischen Räumen} \label{satz:metr-raum-kompaktheit-aequ-charakt-2.2.4} Im metrischen Raum $(X,d)$ sind äquivalent: \begin{enumerate} \item - \index{kompakt!überdeckungs!} + \index{kompakt!überdeckungs-} $K ⊂ X$ ist kompakt (überdeckungskompakt) \item + \index{kompakt!abzählbar} Jede Folge in $K$ besitzt mindestens einen Häufungspunkt in $K$ (abzählbar kompakt) \item \index{kompakt!folgen-} @@ -395,6 +457,49 @@ Nun ein paar Charakterisierungen von kompakten Mengen in metrischen Räumen. \index{Abzählbarkeitsaxiom!zweites} Für „$(b) \Rightarrow (c)$“ benötigt man das erste Abzählbarkeitsaxiom, also die Existenz von abzählbaren Umgebungsbasen für jeden Punkt. \end{bemerkung} +\begin{proof}[\cref{satz:metr-raum-kompaktheit-aequ-charakt-2.2.4}] + $(a) ⇒ (b)$: + Nehmen wir umgekehrt an, es gäbe eine Folge $(x_n)_{n ∈ ℕ}$, die keinen Häufungspunkt besitzt. + Dann gibt es für jedes $y ∈ X$ ein $r_y > 0$, so dass $N_y := \{k ∈ ℕ: x_k ∈ B_{r_y}(y)\}$ endlich ist. + Dann sind die offenen Kugeln $(B_{r_y}(y))_{y ∈ X}$ eine offene Überdeckung von $X$, daher existiert, weil $X$ kompakt ist, $F ⊂ X$ endlich mit $X ⊂ \bigcup_{y ∈ F}B_{r_y}(y)$. + Aber das impliziert schon $ℕ = \bigcup_{y ∈ F} N_y$ im Widerspruch zur Unendlichkeit von $ℕ$. + Somit kann so eine Folge $(x_n)_{n ∈ ℕ}$ ohne Häufungspunkt nicht existieren und $X$ ist abzählbar kompakt. + + $(b) ⇒ (c)$: Sei $(x_n)_{n ∈ ℕ}$ eine Folge in $X$ und $x$ ein Häufungspunkt von $X$. + Wähle nun $n_1 ∈ ℕ$ beliebig und iterativ $n_{k+1} > n_k$ mit $d(x,x_{n_{k+1}}) < 1/k$. Dann ist $(x_{n_k})_{k ∈ }$ eine Teilfolge von $(x_n)_{n ∈ ℕ}$, die gegen $x$ konvergiert. + + $(c) ⇒ (a)$: %Siehe zum Beispiel \cite[Ch 3, Th 28.2]{munkres2000topology}. + Wir zeigen zunächst, dass, wenn $X$ folgenkompakt ist, und $\mathcal A$ eine offene Überdeckung von $X$ ist, ein $δ > 0$ existiert, so dass jede Teilmenge von $X$ mit Durchmesser höchstens $δ$ in einer Menge aus $\mathcal A$ enthalten ist. + Angenommen, es würde kein $δ > 0$ mit dieser Eigenschaft geben. + Dann gibt es insbesondere für jedes $n ∈ ℕ$ eine Menge mit Durchmesser kleiner $1/n$. die nicht in einer Menge aus $\mathcal A$ enthalten ist. + Sei für jede natürliche Zahl $n$ $C_n$ so eine Menge und $x_n ∈ C_n$. + Per Annahme besitzt $(x_n)_{n ∈ ℕ}$ eine konvergente Teilfolge, sagen wir $(x_{n_k})_{k ∈ ℕ}$, die gegen $a ∈ X$ konvergiert. + Dann ist $a$ in einem $A ∈ \mathcal A$ enthalten. + Da $A$ offen ist, gibt es ein $ε > 0$, so dass $B_ε(a) ⊂ A$. + Ist nun $k$ so groß, dass $1/n_k < ε/2$, dann ist $C_{n_k} ⊂ B_{ε/2}(x_{n_k})$. + Aber das ist ein Widerspruch zur Annahme. + + Zweitens zeigen wir, dass, wenn $X$ folgenkompakt ist und $ε > 0$, wir eine endliche Überdeckung von $X$ durch $ε$-Bällen finden können. + Auch hier nehmen wir an, das würde nicht gehen. + Sei $ε > 0$ so, dass $X$ nicht von endlich vielen $ε$-Bällen überdeckt werden kann. + Wir konstruieren nun iterativ eine Folge, die keine konvergente Teilfolge besitzen kann: Sei $x_1 ∈ X$ beliebig. + Da per Wahl von $ε$ $X$ nicht komplett von $B_ε(x_1)$ überdeckt wird, gibt es ein $x_2 ∈ X \setminus B_ε(x_1)$. + Wähle nun iterativ, wenn $x_n$ schon konstruiert ist, $x_{n+1}$ so, dass es nicht in der Vereininung + \[ + B_ε(x_1) ∪ \cdots ∪ B_ε(x_n) + \] + liegt. + Das geht, da nach Annahme $X$ nicht von endlich vielen $ε$-Bällen überdeckt werden kann. + Nach Konstruktion ist nun $d(x_n, x_m) > ε$ für $n \ne m$. + Somit kann $(x_n)_{n ∈ ℕ}$ keine Cauchy"=Teilfolge, also auch keine konvergente Teilfolge enthalten und $X$ ist somit nicht folgenkompakt. + + Nun folgern wir die Behauptung: Sei $\mathcal A$ eine offene Überdeckung von $X$. + Da $X$ folgenkompakt ist, gibt es nun ein $δ > 0$, so dass jede Menge mit Durchmesser kleiner $δ$ in einer Menge in $\mathcal A$ enthalten ist. + Sei nun $ε = δ/3$. + Da $X$ folgenkompakt ist, gibt es eine endliche Überdeckung von $X$ aus $ε$-Bällen. + Da jeder dieser Bälle einen Durchmesser von höchstens $2δ/3$ hat, ist er in einer Menge in $\mathcal A$ enthalten. + Das erlaubt uns, eine endliche Teilüberdeckung aus $\mathcal A$ auszuwählen, die $X$ überdeckt. +\end{proof} \section{Vollständigkeit in metrischen Räumen und der Satz von Baire} \label{sec:vollst-metr-raum} \begin{definition}[Cauchy-Folge] @@ -410,7 +515,7 @@ Nun ein paar Charakterisierungen von kompakten Mengen in metrischen Räumen. Sei etwa $\lim_{n→∞} x_n = x$. Sei $ε > 0$. Da $(x_n)_{n ∈ ℕ}$ gegen $x$ konvergiert, gibt es $N ∈ ℕ$ mit $d(x_n,x)< ε/2$ für alle $n ≥ N$, also mit der Dreiecksungleichung \[ - ∀n,m ≥ N: d(x_n,x_m) ≤ d(x_n,x) + d(x, x_m) < \frac {ε} 2 + \frac {ε} 2 = ε. + ∀n,m ≥ N: d(x_n,x_m) ≤ d(x_n,x) + d(x, x_m) < ε/2 + ε/2 = ε. \] \end{proof} \begin{definition}[vollständiger metrischer Raum] @@ -419,8 +524,12 @@ Nun ein paar Charakterisierungen von kompakten Mengen in metrischen Räumen. \index{vollständig} Der metrische Raum $(X,d)$ heißt \emph{vollständig}, falls jede Cauchy-Folge in $(X,d)$ konvergiert. \end{definition} -Nicht jeder metrische Raum braucht vollständig zu sein (man betrachte hierfür z.B. $ℚ$ und die Folge der Partialsummen der Dezimalbruchentwicklung von $\sqrt 2$), -jedoch lässt sich jeder metrische Raum zu einem vollständigen Erweitern. +\begin{bemerkung-nn} + Nicht jeder metrische Raum braucht vollständig zu sein (man betrachte + hierfür z.B. $ℚ$ und die Folge der Partialsummen der Dezimalbruchentwicklung + von $\sqrt 2$), jedoch lässt sich jeder metrische Raum zu einem + vollständigen Erweitern. +\end{bemerkung-nn} \begin{satz} \label{satz:metr-raum-vervollstd-2.3.4} \index{Vervollständigung} @@ -428,8 +537,11 @@ jedoch lässt sich jeder metrische Raum zu einem vollständigen Erweitern. Dieser Raum $(\tilde X, \tilde d)$ heißt die Vervollständigung von $(X,d)$. \end{satz} \begin{proof} - Zwei Cauchyfolgen $(x_n)_{n ∈ ℕ}$ und $(y_n)_{n ∈ ℕ}$ seien äquivalent, falls $d(x_n,y_n) \xrightarrow[n → \infty ]{} 0$. - Hierdurch ist eine Äquivalenzrelation definiiert. Sei $[(x_n)_{n ∈ ℕ}]$ die vom Repräsententaten $(x_n)_{n ∈ ℕ}$ erzeugte Klasse. Man setzt + Zwei Cauchyfolgen $(x_n)_{n ∈ ℕ}$ und $(y_n)_{n ∈ ℕ}$ seien äquivalent, genau dann, wenn + \[ d(x_n,y_n) \yrightarrow[n → \infty ]{} 0. \] + Hierdurch ist eine Äquivalenzrelation definiiert. + Sei $[(x_n)_{n ∈ ℕ}]$ die vom Repräsententaten $(x_n)_{n ∈ ℕ}$ erzeugte Klasse. + Man setzt \[ \tilde X \coloneq \{ [ (x_n)_{n ∈ ℕ}] : (x_n)_{n ∈ ℕ} \text{ ist Cauchy-Folge in }(X,d)\} \] @@ -449,7 +561,6 @@ jedoch lässt sich jeder metrische Raum zu einem vollständigen Erweitern. \] Die umgekehrte Ungleichung ergibt sich aus Vertauschung der Rollen. Man rechnet leicht nach, dass $(\tilde X, \tilde d)$ ein vollständiger Raum ist. Wir können $(X,d)$ durch die entsprechenden konstanten Folgen isometrisch in $\tilde X$ einbetten. - \todo{Hier fehlt noch was.} \end{proof} \begin{bemerkung-nn} Wendet man diese Technik auf $ℚ$ mit der natürlichen Metrik an, dann erhält man $(ℝ,d)$ als vollständige Hülle. @@ -461,8 +572,10 @@ jedoch lässt sich jeder metrische Raum zu einem vollständigen Erweitern. Sei $(X,d)$ ein vollständiger metrischer Raum und seien $(x_n)_{n * ℕ} ⊂ X$ und $(r_n)_{n ∈ ℕ} ⊂ (0,\infty ) $ Folgen mit der Eigenschaft \begin{enumerate} - \item $\cl B_{r_{n+1}}(x_{n+1}) ⊂ B_{r_n} (x_n)$ - \item $\lim_{n \to \infty } r_n = 0$. + \item + $\cl B_{r_{n+1}}(x_{n+1}) ⊂ B_{r_n} (x_n)$ + \item + $\lim\limits_{n \to \infty } r_n = 0$. \end{enumerate} Dann gibt es genau ein $x_0 ∈ X$ mit $x_0 ∈ \bigcap_{n ∈ ℕ} \cl B_{r_n} (x_n)$. \end{satz} @@ -473,7 +586,7 @@ jedoch lässt sich jeder metrische Raum zu einem vollständigen Erweitern. \] Also \[ - d(x_{n+p},x_n) \le r_n \xrightarrow[n → \infty ]{} 0. + d(x_{n+p},x_n) \le r_n \yrightarrow[n → \infty ]{} 0. \] Damit ist $(x_n){n ∈ ℕ}$ eine Cauchyfolge und damit konvergiert gegen ein $x_0 ∈ X$, da $X$ vollständig ist. Außerdem gilt @@ -488,7 +601,7 @@ jedoch lässt sich jeder metrische Raum zu einem vollständigen Erweitern. Für die Eindeutigkeit sei $\tilde x_0$ ebenfalls in $\bigcap_{n ∈ ℕ} \cl B_{r_n}(x_n)$. Dann folgt \[ - d(x_0,\tilde x_0) \le \underbrace{d(x_0,x_n)}_{\le r_n} + \underbrace{d(x_n, \tilde x_0)}_{\le r_n} \le 2r_n \xrightarrow[n → \infty ]{} 0. + d(x_0,\tilde x_0) \le \underbrace{d(x_0,x_n)}_{\le r_n} + \underbrace{d(x_n, \tilde x_0)}_{\le r_n} \le 2r_n \yrightarrow[n → \infty ]{} 0. \] Doch damit war bereits $x_0 = \tilde x_0$. \end{proof} @@ -524,7 +637,7 @@ Der folgende Satz wird beim Beweis mehrerer fundamentaler Sätze benötigt, z.B B_{r_2}(x_2) ⊂ B_{r_1/2} (x_1) \] und $B_{r_2}(x_2) ∩ M_2 = \emptyset$. - Durch Fortsetzen dieses Schemas finden wir eine Folge $(x_n)_{n ∈ ℕ} ⊂ X$ und Radien $(r_n)_{n ∈ ℕ} ⊂ (0,\infty )$ mit $r_n \le r/2^n \xrightarrow[n → \infty ]{} 0$. + Durch Fortsetzen dieses Schemas finden wir eine Folge $(x_n)_{n ∈ ℕ} ⊂ X$ und Radien $(r_n)_{n ∈ ℕ} ⊂ (0,\infty )$ mit $r_n \le r/2^n \yrightarrow[n → \infty ]{} 0$. Damit sind alle Voraussetzungen von~\cref{satz:schachtelsatz-2.3.5} erfüllt. Folglich existiert genau ein \[ \tilde x ∈ \bigcap_{n ∈ ℕ} B_{r_n} (x_n) ⊂ B_r(x_0) ⊂ M. @@ -562,7 +675,7 @@ Der Vollständigkeit halber noch eine alternative (stärkere) Formulierung: \item $\cl{B_{r_n}(x_n)} ⊂ U_n ∩ B_{r_{n-1}} (x_{n-1})$ \end{enumerate} - Dazu beachte man, dass $U_n ∩ B_{r_{n-1}}(x_{n-1})$ nichtleer und offen ist, also existiert $x_n$ und $\frac 1 n > \epsilon > 0$ mit $B_\epsilon (x_n) ⊂ U_n ∩ B_{r_{n-1}} (x_{n-1})$ und $r_n = \frac \epsilon 2$ ist wie gewünscht. + Dazu beachte man, dass $U_n ∩ B_{r_{n-1}}(x_{n-1})$ nichtleer und offen ist, also existiert $x_n$ und $1/n > \epsilon > 0$ mit $B_\epsilon (x_n) ⊂ U_n ∩ B_{r_{n-1}} (x_{n-1})$ und $r_n = \epsilon/2$ ist wie gewünscht. Für $m \ge n$ impliziert (ii), dass $x_m ∈ B_{r_n}(x_n)$ und aus (i) folgt, dass die Folge $(x_n)_{n ∈ ℕ}$ damit eine Cauchyfolge ist. Damit konvergiert $(x_n)_{n ∈ ℕ}$ gegen ein $x ∈X$. Sei nun $N ∈ ℕ$ und $m > N$. @@ -611,5 +724,5 @@ Der Vollständigkeit halber noch eine alternative (stärkere) Formulierung: %%% Local Variables: %%% mode: latex -%%% TeX-master: "funkana-ebook" +%%% TeX-master: "funkana" %%% End: diff --git a/ch03-topologisch-lineare-raeume.tex b/ch03-topologisch-lineare-raeume.tex index f55914a..bbf5265 100644 --- a/ch03-topologisch-lineare-raeume.tex +++ b/ch03-topologisch-lineare-raeume.tex @@ -20,30 +20,49 @@ Erklärtes Ziel dieses Kapitels wird sein, die beiden Strukturen aus den vorheri Sei $X$ ein linearer Raum über $\K$. Die Abbildung $\norm\cdot: X → [0,\infty )$ heißt \emph{Norm} auf $X$, falls für alle $x, y ∈ X, \alpha ∈ K$ gilt: \begin{enumerate} - \item $\norm x = 0 \Longleftrightarrow x = 0$ (Definitheit) \item + \index{Definitheit} + $\norm x = 0 \Longleftrightarrow x = 0$ (Definitheit) + \item + \index{Homogenität} $\norm{\alpha x} = |\alpha | \norm x$ (Homogenität) \item + \index{Dreiecksungleichung} $\norm{x+y} \le \norm x + \norm y$ (Dreiecksungleichung) \end{enumerate} $(X,\norm\cdot)$ heißt dann \emph{normierter Raum}. \end{definition} -\begin{bemerkung} +\begin{definition}[Normtopologie] + \label{defi:normtopologie-3.1.2} + \index{Normtopologie} Durch $d(x,y) \coloneq \norm{x-y}$ wird ein normierter Raum auch ein metrischer, also insbesondere auch ein topologischer Raum. Diese induzierte Topologie auf $(X, \norm\cdot)$ heißt \emph{Normtopologie}. +\end{definition} - Ohne die lineare Struktur macht der normierte Raum gar keinen Sinn, da für die Definition einiger der Normaxiome die Vektorraumoperationen verwendet werden. -\end{bemerkung} +\begin{bemerkung-nn} + Man kann nur Normen auf linearen Räumen definieren, denn ohne die lineare Struktur macht der normierte Raum gar keinen Sinn, da für die Definition der Normaxiome die Vektorraumoperationen verwendet werden. +\end{bemerkung-nn} + +\begin{lemma-nn} + \index{Dreiecksungleichung} + \label{bem:umgekehrte-dreicksungleichung} + In einem normierten Raum $(X,\norm -)$ gilt die \emph{umgekehrte Dreiecksungleichung} + \[ + ∀x,y ∈ X: \big| \snorm x - \snorm y \big| \le \norm{x+y}. + \] +\end{lemma-nn} \begin{beispiele} \begin{enumerate} \item + \index{$ℝ^n$} Betrachte den $ℝ^n$ mit $\norm x _{p} \coloneq \left( \sum_{i=1}^n |x_i|^p \right)^{1/p}$ mit $1 \le p < \infty $ ist ein normierter Raum, genauso wie mit $\norm{x}_{\infty } \coloneq \max_{1 \le i \le n} |x_i|$. Insbesondere gibt es im $ℝ^n$ überabzählbar viele verschiedene Normen. Wir werden jedoch später sehen, dass diese Normen alle die gleiche Topologie erzeugen. \item + \index{$C[a,b]$} Der Raum aller stetigen Funktionen auf einem kompaktem Intervall $C[a,b]$ mit $\norm{x}_{\infty } \coloneq \max_{t ∈ [a,b]} |x(t)|$ ist ein normierter Raum. Außerdem wird durch \[ @@ -51,18 +70,21 @@ Erklärtes Ziel dieses Kapitels wird sein, die beiden Strukturen aus den vorheri \] ebenfalls eine Norm definiert. \item + \index{$C(\cl{Ω})$} Sei $\Omega ⊂ ℝ^n$ offen und beschränkt. Dann wird $C(\cl{\Omega})$ mit \[ \norm{x}_{\infty } \coloneq \max_{t ∈ \cl \Omega} |x(t)| \] auch zu einem normierten Raum. \item + \index{$L^p$} $L^p(\Omega) = \L^p(\Omega)/\mathcal N$, wobei $\mathcal N = \{ f: \Omega → \R, f(t) = 0 \text{ fast überall}\}$ ist mit \[ \norm x \coloneq \left(∫_{\Omega} |x(t)|^p dt \right)^{1/p} \] ein normierter Raum, wobei $1 \le p < \infty $. \item + \index{$\ell^p$} $\ell^p$ mit \[ \norm x _{p} \coloneq \left( \sum_{i=1}^n |x_i|^p \right)^{1/p} @@ -72,6 +94,7 @@ Erklärtes Ziel dieses Kapitels wird sein, die beiden Strukturen aus den vorheri \end{beispiele} \begin{lemma} + \label{lemma:normierter-raum-ist-top-vr-3.1.4} Sei $(X, \norm\cdot)$ ein normierter Raum. Dann sind die Abbildungen $+$, $\cdot$ und $\norm\cdot$ stetig. \end{lemma} \begin{proof} @@ -85,20 +108,24 @@ Erklärtes Ziel dieses Kapitels wird sein, die beiden Strukturen aus den vorheri \] und \[ - |\snorm{x_n} - \norm{x}| \le \norm{x_n - x} + |\snorm{x_n} - \norm{x}| \le \snorm{x_n - x} \] nach der umgekehrten Dreiecksungleichung. Folglich sind die zu betrachtenden Abbildungen alle folgenstetig, und, da metrische Räume stets dem ersten Abzähhlbarkeitsaxiom genügen, auch stetig. \end{proof} +Die Wichtigkeit dieser Eigenschaft wollen wir in diesem Korrolar betonen: + \begin{korollar} + \label{kor:normierter-raum-ist-top-vr-3.1.5} Jeder normierte Raum versehen mit der Normtopologie ist ein topologischer linearer Raum. Deshalb ist auch keine Unterscheidung zwischen normierten Räumen und normierten topologischen linearen Räumen nötig. \end{korollar} \section{Topologische lineare Räume} +\label{sec:topol-line-raume} \begin{bemerkung-nn} - Hierbei sei stets die Topologie von $X×X$ die Produktopologie, bei den Körpern $\K = \begin{cases} ℝ \\ ℂ \end{cases}$ die übliche Topologie. + Hierbei sei stets die Topologie von $X×X$ die Produktopologie, bei den Körpern $ℝ$ und $ℂ$ die übliche Topologie. Wir schreiben im Folgenden für Mengen $M, M_1, M_2 ⊂ X$ und $\alpha ⊂ \K$ nun \[ M_1 + M_2 \coloneq s(M_1,M_2) \coloneq \{x+y : x ∈ M_1, y ∈ M_2\}, @@ -109,76 +136,101 @@ Erklärtes Ziel dieses Kapitels wird sein, die beiden Strukturen aus den vorheri \end{bemerkung-nn} \begin{lemma} + \label{lemma:top-raum-mit-lin-struktur-aeq-charak-stetigkeit-addition-3.2.1} Hat der topologische Raum $(X,\T)$ auch eine lineare Struktur, so sind äquivalent: \begin{enumerate} - \item Die Addition $s$ ist stetig. \item - Für beliebiges $x, y ∈ X$ gilt: Zu jeder Umgebung $O_{x+y} ∈ \T$ existieren Umgebungen $O_x ∈ \T$ von $x$ und $O_y ∈ \T$ von $y$ mit $O_x + O_y ⊂ O_{x+y}$ + Die Addition $s$ ist stetig. + \item + Für beliebiges $x, y ∈ X$ gilt: Zu jeder Umgebung $O_{x+y} ∈ \T$ von $x+y$ existieren Umgebungen $O_x ∈ \T$ von $x$ und $O_y ∈ \T$ von $y$ mit $O_x + O_y ⊂ O_{x+y}$ \end{enumerate} \end{lemma} \begin{proof} $s$ ist stetig in $(x,y)$ genau dann, wenn zu jeder Umgebung $O_{x,y} ∈ \T_X$ - von $(x,y)$ existiert eine Umgebung $U ⊂ \T_{X×X}$ von $(x,y)$ mit $s(U) ⊂ O_{x+y}$. + von $(x,y)$ eine Umgebung $U ⊂ \T_{X×X}$ von $(x,y)$ mit $s(U) ⊂ O_{x+y}$ existiert. Nach Definition der Produkttopologie existieren dann Umgebungen $O_x ∈ \U_x$ und $O_y ∈ \U_y$ mit $O_x × O_y ⊂ U$. Damit ist \[ O_x + O_y = s(O_x, O_y) = s(O_x × O_y) ⊂ s(U) ⊂ O_{x+y}. \] - Analog zeigt man die entsprechende Aussage für die skalare Multiplikation: \end{proof} + Analog zeigt man die entsprechende Aussage für die skalare Multiplikation: \begin{lemma} + \label{lemma:top-raum-mit-lin-struktur-aeq-charak-stetigkeit-multiplikation-3.2.2} Hat der topologische Raum $(X,\T)$ auch eine lineare Struktur, so sind äquivalent: \begin{enumerate} - \item Die Addition $m$ ist stetig. \item - Für beliebiges $\alpha ∈ \K, x ∈ X$ gilt: Zu jeder Umgebung $O_{\alpha x} ∈ \T$ existieren Umgebungen $O_x ∈ \T$ von $x$ und $O_\alpha ∈ \T$ von $y$ mit $O_\alpha × O_x ⊂ O_{\alpha x}$. + Die Multiplikation $m$ ist stetig. + \item + Für beliebiges $\alpha ∈ \K, x ∈ X$ gilt: Zu jeder Umgebung $O_{\alpha x} ∈ \T$ von $αx$ existieren Umgebungen $O_x ∈ \T$ von $x$ und $O_\alpha ∈ \T$ von $y$ mit $O_\alpha × O_x ⊂ O_{\alpha x}$. \end{enumerate} \end{lemma} +\begin{noproof} + ~ +\end{noproof} +Sei nun $X$ ein topologischer linearer Raum, das heißt, insbesondere ist die Multiplikation stetig. Betrachtet man insbesondere die Stetigkeit am Punke $\alpha =0$ und $x ∈ X$ beliebig, dann gilt also: -Für jede Umgebung $O_0 ∈ \U_0 ⊂ X$ existiert eine Umgebung $O_x ∈ \U_x$ und ein $r > 0$, so dass +Für jede Umgebung $O_0 ∈ \U_0 ⊂ \Pot X$ existiert eine Umgebung $O_x ∈ \U_x$ und ein $r > 0$, so dass \[ ∀β: |β| 0 ∃ \delta > 0 ∃ r> 0 ∀β ∈ \K ∀y ∈ X: + ∀\epsilon > 0\; ∃ \delta > 0\; ∃ r> 0\; ∀β ∈ \K\;∀y ∈ X: \begin{rcases} |β - \alpha | < r \\ d(x,y) < \delta \end{rcases} \implies d(βy,\alpha x) < \epsilon \] - +\end{bemerkung-nn} \begin{lemma} - \label{lemma-metrischer-linearer-raum-charak} + \label{lemma:metrischer-linearer-raum-charak-3.4} Sei $(X,d)$ ein metrischer Raum mit linearer Struktur und mit einer translationinvarianten Metrik. Dann ist $X$ mit der von $d$ erzeugten Topologie ein \emph{metrischer linearer Raum} genau dann, wenn für alle $\alpha ∈ \K, x ∈ X$ und beliebige Nullfolgen $(x_n)_{n ∈ ℕ} ⊂ X, (\alpha _n)_{n ∈ ℕ)} ⊂ \K$ gilt \begin{gather*} - \alpha x_n \xrightarrow[n → \infty ]{} 0 \\ - \alpha x_n \xrightarrow[n → \infty ]{} 0 \\ - \alpha _nx_n \xrightarrow[n → \infty ]{} 0 + \alpha x_n \yrightarrow[n → \infty ]{} 0 \\ + \alpha x_n \yrightarrow[n → \infty ]{} 0 \\ + \alpha _nx_n \yrightarrow[n → \infty ]{} 0 \end{gather*} \end{lemma} \begin{proof} @@ -243,10 +305,10 @@ Für die Stetigkeit der skalaren Multiplikation im Punkt $(\alpha ,x) ∈ \K × „$⇐$“: Wegen der Äquivalenz von Stetigkeit und Folgenstetigkeit ist zu zeigen \[ \begin{rcases} - \alpha _n \xrightarrow[n → \infty ]{} \alpha ∈ \K \\ - x_n \xrightarrow[n → \infty ]{} x ∈ X + \alpha _n \yrightarrow[n → \infty ]{} \alpha ∈ \K \\ + x_n \yrightarrow[n → \infty ]{} x ∈ X \end{rcases} - \implies \alpha _n x_n \xrightarrow[n → \infty ]{} \alpha x. + \implies \alpha _n x_n \yrightarrow[n → \infty ]{} \alpha x. \] Sei dazu $z_n \coloneq x_n - x ∈ X$, $γ_n \coloneq \alpha _n - \alpha ∈ \K$. Dann ist @@ -258,37 +320,38 @@ Für die Stetigkeit der skalaren Multiplikation im Punkt $(\alpha ,x) ∈ \K × \begin{align*} d(\alpha _nx_n,\alpha x) &= d(\alpha nx_n - \alpha x,0) = d(γ_nz_n + γnx + \alpha z_n, 0) \\ &\le \underbrace{d(γ_nz_n,0)}_{→ 0} + \underbrace{d(γ_nx, 0)}_{→ 0} + -\underbrace{d(\alpha z_n, 0)}_{→ 0} \xrightarrow[n → \infty]{} 0. +\underbrace{d(\alpha z_n, 0)}_{→ 0} \yrightarrow[n → \infty]{} 0. \end{align*} Da die Addition ohnehin immer stetig ist, sind wir fertig. \end{proof} - - -\begin{definition} +\begin{definition}[Quasi-Norm] + \index{Quasi-Norm} + \index{Raum!quasi-normierter} + \label{defi:quasinorm-3.3.5} Eine Abbildung $|\cdot|: X → [0,\infty )$ heißt \emph{Quasi-Norm} auf dem Linearen Raum $X$, falls gilt: - \begin{enumerate}[label=(Q\arabic*)] + \begin{wenumerate}[label=(Q\arabic*)] \item - $|x| \ge 0$ für alle $x ∈ X$ und $|x| = 0$ genau dann, wenn $x = 0$. + $|x| \ge 0$ für alle $x ∈ X$ und $|x| = 0$ genau dann, wenn $x = 0$ (positiv definit) \item $|-x| = |x|$ für alle $x ∈ X$ \item - $|x+y| \le |x| + |y|$ für alle $x,y ∈ X$ + $|x+y| \le |x| + |y|$ für alle $x,y ∈ X$ (Dreiecksungleichung)\index{Dreiecksungleichung} \item - $|\alpha x_n| \xrightarrow[n → \infty ]{} 0$ für $\alpha ∈ \K$, falls $|x_n| → 0$ + $|\alpha x_n| \yrightarrow[n → \infty ]{} 0$ für $\alpha ∈ \K$, falls $|x_n| → 0$ \item - $|\alpha _nx| \xrightarrow[n → \infty ]{} 0$ für $x ∈ X$, falls $|\alpha _n| → 0$ + $|\alpha _nx| \yrightarrow[n → \infty ]{} 0$ für $x ∈ X$, falls $|\alpha _n| → 0$ \item - $|\alpha _nx_n| \xrightarrow[n → \infty ]{} 0$ falls $|x_n| → 0$ und $|\alpha _n| → 0$ - \end{enumerate} + $|\alpha _nx_n| \yrightarrow[n → \infty ]{} 0$ falls $|x_n| → 0$ und $|\alpha _n| → 0$ + \end{wenumerate} $(X,|\cdot|)$ heißt dann \emph{quasi-normierter} Raum. \end{definition} - \begin{bemerkung} - Jeder normierte Raum ist auch ein quasi-normierter Raum. + \label{bem:norm-raum-ist-quasinorm-raum-3.3.6} + Jeder normierte Raum ist auch ein quasi-normierter Raum mit $|-| := \norm-$. \end{bemerkung} - \begin{satz} + \label{satz:quasi-norm-ind-transinvar-metrik-3.3.7} \begin{enumerate} \item Ist $|\cdot|$ eine Quasi-Norm auf $X$, so wird durch $d(x,y) \coloneq |x-y|$ eine translationsinvariante Metrik definiert, welche $X$ zu einem metrischen linearen Raum macht. @@ -298,13 +361,15 @@ Für die Stetigkeit der skalaren Multiplikation im Punkt $(\alpha ,x) ∈ \K × \end{enumerate} \end{satz} \begin{proof} - Das folgt direkt aus den Axiomen und \cref{lemma-metrischer-linearer-raum-charak}. + Das folgt direkt aus den Axiomen und~\cref{lemma:metrischer-linearer-raum-charak-3.4} \end{proof} - - -Speziell für die Anwendung sehr wichtige metrische lineare Räume werden von Semi-Normen erzeugt. - -\begin{definition} +\begin{bemerkung-nn} + Speziell für die Anwendung sehr wichtige metrische lineare Räume werden von Semi-Normen erzeugt. +\end{bemerkung-nn} +\begin{definition}[Semi-Norm] + \index{Semi-Norm} + \index{Raum!semi-normierter} + \label{defi:seminorm-3.3.8} Sei $X$ ein linearer Raum. Eine Abbildung $p: X → ℝ$ heißt \emph{Semi-Norm} oder \emph{Halbnorm}, falls folgendes gilt: \begin{enumerate}[label=(S\arabic*)] @@ -313,21 +378,26 @@ Speziell für die Anwendung sehr wichtige metrische lineare Räume werden von Se \item $∀ x ∈ X, \alpha ∈ \K: p(\alpha x) = |\alpha | p(x)$ (Homogenität) \item + \index{Dreiecksungleichung} $∀ x, y ∈ X: p(x+y) \le p(x) + p(y)$ (Dreiecksungleichung) \end{enumerate} $(X,p)$ heißt dann \emph{semi-normierter} Raum. \end{definition} - \begin{beispiel-nn} - $\L^p(\Omega)$ ist ein semi-normierter Raum. + \index{$\L^p(Ω)$} + Sei $Ω ⊂ ℝ^n$, $p > 1$. + Dann ist $\L^p(\Omega)$ ist ein semi-normierter Raum. \end{beispiel-nn} \begin{bemerkung} + \label{bem:seminorm-ind-norm-auf-faktorraum-3.9} Jeder semi-normierte Raum $(X,p)$ erzeugt einen normierten Raum $(X/N,p)$, wobei $N = \{ x ∈ X: p(x) = 0\}$ ein linearer Unterraum ist. \end{bemerkung} \begin{satz} \label{satz-abzaehlbares-prod-seminormierter-raeume} + \label{satz:abzaehbar-viele-seminormen-transinvar-metrik-3.3.10} + \index{Semi-Norm!abzählbar viele} Es seien $p_n: X → ℝ, n ∈ ℕ$ abzählbar viele Semi-Normen auf einem linearen Raum mit der Eigenschaft \begin{equation} p_n(x) = 0 \text{ für alle n ∈ ℕ } \implies x = 0. \label{eq:seminorm-folge-blub} @@ -338,13 +408,15 @@ Speziell für die Anwendung sehr wichtige metrische lineare Räume werden von Se \] eine translationsinvariante Metrik auf $X$, welche $X$ zum metrischen linearen Raum macht. \end{satz} +\begin{noproof} + ~ +\end{noproof} \begin{bemerkung} $p_n: X → ℝ$ sind auf $(X,d)$ stetig. Das folgt aus (für $x_i → x_0$ in $X$) \[ - |p_n(x_i) - p_n(x_0)| \le p_n(x_i-x_0) \xrightarrow{} 0 + |p_n(x_i) - p_n(x_0)| \le p_n(x_i-x_0) \yrightarrow{} 0. \] - und einer Übungsaufgabe. \end{bemerkung} \begin{satz} @@ -388,7 +460,6 @@ Speziell für die Anwendung sehr wichtige metrische lineare Räume werden von Se \[ \sum_{n=n_0}^\infty 2^{-n} < \frac r 2. \] - mit $\epsilon \coloneq \frac r 2 $ gilt dann \[ \bigcap_{n=1}^{n_0} U(p_(,\epsilon ) ⊂ B_r(0). @@ -402,6 +473,7 @@ Speziell für die Anwendung sehr wichtige metrische lineare Räume werden von Se \end{proof} \begin{bemerkung} + \label{bem:abz-viele-seminormen-lokalkonvex-3.3.13} Die Mengen $U(p_n,\epsilon _n)$ und deren endlichen Schnitte sind konvexe Mengen, das heißt \[ x, y ∈ U(p_n,\epsilon _n),\alpha ∈ [0,1] \implies \alpha x+(1-\alpha )y ∈ U(p_n,\epsilon _n) @@ -413,14 +485,15 @@ Speziell für die Anwendung sehr wichtige metrische lineare Räume werden von Se p_n(\alpha x + (1-\alpha )y) \le |\alpha | \underbrace{p_n(x)}_{< \epsilon _n} + |1-\alpha |\underbrace{p_n(y)}_{< \epsilon _n} = \epsilon _n. \] \end{proof} - -Also besitzt der in \cref{satz-abzaehlbares-prod-seminormierter-raeume} gewonne metrische lineare Raum $(X,d)$ eine Umgebungsbasis von $0$, die nur aus konvexen elementen besteht. - -\begin{definition} +Also besitzt der in \cref{satz-abzaehlbares-prod-seminormierter-raeume} gewonne metrische lineare Raum $(X,d)$ eine Umgebungsbasis der $0$, die nur aus konvexen Elementen besteht. +\begin{definition}[lokal-konvex] + \index{lokal-konvex} + \label{defi:lokalkonvex-3.3.14} Ein topologischer linearer Raum $(X,\T)$, in dem jedes $x ∈ X$ eine Umgebungsbasis besitzt, die nur aus konvexen Mengen besteht, heißt \emph{lokalkonvex}. \end{definition} \begin{satz} + \label{satz:seminormen-lokal-konvexer-t2-raum-3.3.15} Sei $X$ ein linearer Raum mit Semi-Normen $p_i, i ∈ I$, wobei $I$ eine beliebige Indexmenge ist, mit der Eigenschaft \[ p_i(x) = 0 \text { für alle } i ∈ I \implies x = 0. @@ -437,28 +510,33 @@ Also besitzt der in \cref{satz-abzaehlbares-prod-seminormierter-raeume} gewonne Wir werden die unten angegebenen Beispiele auch gleich auf Vollständigkeit untersuchen. \begin{definition} + \label{defi:frechet-raum-banach-raum:3.4.1} + \index{Raum!Banach-} + \index{Raum!Fréchet-} + \index{Fréchetraum} + \index{Banachraum} \begin{enumerate} \item - Ein metrischer linearer Raum $(X,d)$ der vollständig ist, heißt \emph{Fréchet"=Raum}. + Ein metrischer linearer Raum $(X,d)$ der vollständig ist, heißt \emph{Fréchetraum}. \item - Ein normierter Raum $(X,\norm\cdot)$, der vollständig ist, heißt \emph{Banach"=Raum}. - + Ein normierter Raum $(X,\norm\cdot)$, der vollständig ist, heißt \emph{Banachraum}. \end{enumerate} \end{definition} -\begin{beispiel-nn}[$\ell^p$-Räume] - \begin{enumerate} - \item - $(\ell^p,\norm\cdot_p)$, $1 \le p < \infty $ ist normierter Raum mit - \[ - \norm x _p = \left( \sum_{i=1}^\infty |x_i|^p \right)^{1/p}. - \] - \item - $(\ell^\infty ,\norm\cdot_\infty)$, ist normierter Raum mit $\norm x _\infty = \sup_{i ∈ ℕ} |x_i|$. - \item - $(\ell^p,|\cdot|_p = \norm\cdot_p^p)$, $0 \le p < 1$ ist quasi-normierter Raum. - \end{enumerate} -\end{beispiel-nn} +\subsection{Die Folgenräume \(\ell^p\)} +\label{sec:ellp-raume} +\index{$\ell^p$} +\begin{enumerate} +\item + $(\ell^p,\norm\cdot_p)$, $1 \le p < \infty $ ist normierter Raum mit + \[ + \norm x _p = \left( \sum_{i=1}^\infty |x_i|^p \right)^{1/p}. + \] +\item + $(\ell^\infty ,\norm\cdot_\infty)$, ist normierter Raum mit $\norm x _\infty = \sup_{i ∈ ℕ} |x_i|$. +\item + $(\ell^p,|\cdot|_p = \norm\cdot_p^p)$, $0 \le p < 1$ ist quasi-normierter Raum. +\end{enumerate} \begin{bemerkung} Für $0 < p < q \le \infty $ gilt $\ell^p ⊂ \ell^q ⊂ \ell^\infty $. @@ -476,7 +554,7 @@ Wir werden die unten angegebenen Beispiele auch gleich auf Vollständigkeit unte \end{satz} \begin{proof} Nur für $1 \le p < \infty $. - Sei dazu $(x_n)_{n ∈ ℕ} ⊂ \ell^p$ eine Cauchy-Folge, also + Sei dazu $(x_n)_{n ∈ ℕ} ⊂ \ell^p$ eine Cauchy"=Folge, also $x_n = (ξ_k^n)_{k ∈ ℕ}$ und für jedes $\epsilon > 0$ gibt es ein $n_0$ mit \[ ∀n,m > n_0: \norm{x_n-x_m}_p = \left( \sum_{k=1}^\infty |ξ_k^n-ξ_k^m|^p \right)^{1/p} < \epsilon . @@ -488,7 +566,7 @@ Wir werden die unten angegebenen Beispiele auch gleich auf Vollständigkeit unte Es gilt \[ - \norm{x_n}_! \le \underbrace{\norm{x_n-x_{n_0}}}_{< \epsilon } + \norm{x_{n_0}} \quad \forall n \ge n_0 + \norm{x_n}_p \le \underbrace{\norm{x_n-x_{n_0}}}_{< \epsilon } + \norm{x_{n_0}} \quad \forall n \ge n_0 \] Deshalb existiert ein $M > 0$ mit $\norm{x_n}_p < M$ für alle $n ∈ ℕ$, also \[ @@ -515,48 +593,51 @@ Wir werden die unten angegebenen Beispiele auch gleich auf Vollständigkeit unte \] also die Konvergenz. \end{proof} -\begin{beispiel-nn} - Betrachte den Folgenraum $S = \K^\infty = \{x = (ξ_n)_{n ∈ ℕ}, ξ_n ∈ \K\}$. - Dann ist - \[ - p_n(x) \coloneq |ξ_n|, \quad p_n: \K^\infty → ℝ - \] - eine abzählbare Familie von Halbnormen mit - \[ - p_n(x) = 0 ∀n ∈ ℕ \implies x = 0 ∈ \K^\infty - \] - Nach \cref{satz-abzaehlbares-prod-seminormierter-raeume} folgt, dass $(\K^\infty , d)$ mit - \[ - d(x,y) \coloneq \sum_{n ∈ ℕ} 2^{-n} \frac{p_n(x-y)}{1+p_n(x-y)} - \] - ein metrischer linearer Raum ist. - Der Konvergenzbegriff entspricht gerade der komponentenweisen Konvergenz, das heißt, für eine Folge $(x_k)_{k ∈ ℕ}$ mit $x_k = (ξ^k_n)_{n ∈ ℕ}$ gilt - \begin{align*} - x_k \xrightarrow[k→\infty ]{} 0 - &\gdw d(x_n,0) \xrightarrow[k→\infty ]{} 0 \\ - &\gdw p_n(x_k) \xrightarrow[k→\infty ]{} ∀ n ∈ ℕ \\ - &\gdw |ξ_n^k| \xrightarrow[k→\infty ]{} 0 ∀ n ∈ ℕ. - \end{align*} - Wir fragen uns nun, ob auf dem $\K^\infty $ auch eine Topologie existiert, so dass der induzierte Konvergenzbegriff der der gleichmäßigen Konvergenz in allen Komponenten entspricht. - Also - \[ - x_k \xrightarrow[k → \infty ]{\text{glm}} 0 ∈ \K^\infty \gdw ∀\epsilon > 0 ∃ k_0 ∈ ℕ: |ξ_n^k| < \epsilon ∀ k \ge k_0 ∀n ∈ ℕ. - \] - Wenn $\K^\infty $ ein topologischer linearer Raum sein soll, ist das nicht möglich. Notwendig wäre, dass für eine Folge $x  ∈ \K^\infty $ - \[ - \alpha _k \xrightarrow[k → \infty ]{} 0 \text{ in } \K \implies \alpha _k x \xrightarrow[k→\infty ]{} \text{ in } X = \K^\infty . - \] - Wähle dazu die Nullfolge $(\alpha _k)_{k ∈ ℕ} = (1/k)_{k ∈ ℕ} ⊂ ℝ$, $x= (n)_{n ∈ ℕ} ⊂ X$. Dann ist - \[ - \alpha _k x = (n/k)_{n ∈ ℕ} ∈ \K^\infty - \] - zwar eine Nullfolge in $\K^\infty$ ist, diese Konvergenz ist jedoch nicht gleichmäßig in $n$. - Man kann zeigen, dass $\K^\infty $ mit $d$ vollständig, also ein Fréchet-Raum, ist. - Ist $\K^\infty $ auch normierbar? - Also gibt es auf $\K^\infty $ eine Norm, welche die gleiche Topologie erzeugt wie die $d$? - Auch das ist nicht möglich: -\end{beispiel-nn} +\subsection{Der Folgenraum $\mathcal S = \K^∞$} +\label{sec:der-folg-mathc} +Wir bezeichnen den Raum $\{x = (ξ_n)_{n ∈ ℕ}, ξ_n ∈ \K\}$ aller Folgen in $\K$ mit $\mathcal S$ oder $\K^\infty$. +\index{$\K^∞$} +\index{$\mathcal S$} +Dann ist \[ + p_n(x) \coloneq |ξ_n|, \quad p_n: \K^\infty → ℝ +\] +eine abzählbare Familie von Halbnormen mit +\[ + p_n(x) = 0 ∀n ∈ ℕ \implies x = 0 ∈ \K^\infty +\] +Nach \cref{satz-abzaehlbares-prod-seminormierter-raeume} folgt, dass $(\K^\infty , d)$ mit +\[ + d(x,y) \coloneq \sum_{n ∈ ℕ} 2^{-n} \frac{p_n(x-y)}{1+p_n(x-y)} +\] +ein metrischer linearer Raum ist. +Der Konvergenzbegriff entspricht gerade der komponentenweisen Konvergenz, das heißt, für eine Folge $(x_k)_{k ∈ ℕ}$ mit $x_k = (ξ^k_n)_{n ∈ ℕ}$ gilt +\begin{align*} + x_k \yrightarrow[k→\infty ]{} 0 + &\gdw d(x_n,0) \yrightarrow[k→\infty]{} 0 \\ + &\gdw p_n(x_k) \yrightarrow[k→\infty]{}0\; ∀ n ∈ ℕ \\ + &\gdw |ξ_n^k| \yrightarrow[k→\infty]{}0\;∀ n ∈ ℕ. +\end{align*} + +Wir fragen uns nun, ob auf dem $\K^\infty $ auch eine Topologie existiert, so dass der induzierte Konvergenzbegriff der der gleichmäßigen Konvergenz in allen Komponenten entspricht. +Also +\[ + x_k \yrightarrow[k → \infty ]{\text{glm}} 0 ∈ \K^\infty \gdw ∀\epsilon > 0 ∃ k_0 ∈ ℕ: |ξ_n^k| < \epsilon ∀ k \ge k_0 ∀n ∈ ℕ. +\] +Wenn $\K^\infty $ ein topologischer linearer Raum sein soll, ist das nicht möglich. Notwendig wäre, dass für eine Folge $x  ∈ \K^\infty $ +\[ + \alpha _k \yrightarrow[k → \infty ]{} 0 \text{ in } \K \implies \alpha _k x \yrightarrow[k→\infty ]{} \text{ in } X = \K^\infty . +\] +Wähle dazu die Nullfolge $(\alpha _k)_{k ∈ ℕ} = (1/k)_{k ∈ ℕ} ⊂ ℝ$, $x= (n)_{n ∈ ℕ} ⊂ X$. Dann ist +\[ + \alpha _k x = (n/k)_{n ∈ ℕ} ∈ \K^\infty +\] +zwar eine Nullfolge in $\K^\infty$ ist, diese Konvergenz ist jedoch nicht gleichmäßig in $n$. +Man kann zeigen, dass $\K^\infty $ mit $d$ vollständig, also ein Fréchet-Raum, ist. +Ist $\K^\infty $ auch normierbar? +Also gibt es auf $\K^\infty $ eine Norm, welche die gleiche Topologie erzeugt wie die $d$? +Auch das ist nicht möglich: + \begin{lemma} \label{lemma-s-metrikkugeln-enthalten-unterraeme} In $(\K^\infty ,d)$ gilt: @@ -605,15 +686,17 @@ Das heißt, was bereits $\alpha = 0$ impliziert. Das ist ein Widerspruch. -\begin{beispiel-nn}[Räume beschränkter Funktionen] - Sei $S$ eine beliebige Menge und $B(S) \coloneq \{ f: S → \K, f(s)$ ist beschränkt $\}$. - Dann wird $B(S)$ mit - \[ - \norm f _{B(S)} \coloneq \sup_{x ∈ S} |f(x)| < \infty , - \] - der $\sup$-Norm, zu einem Banachraum. - Dabei ist offensichtlich, dass $\norm\cdot_{B(S)}$ tatsächlich eine Norm ist, und wir werden in einer Übung zeigen, dass die induzierte Metrik tatsächlich vollständig ist. -\end{beispiel-nn} +\subsection{Räume beschränkter Funktionen} +\label{sec:raume-beschr-funkt} +\index{Raum!beschränkter Funktionen} +\index{$B(S)$} +Sei $S$ eine beliebige Menge und $B(S) \coloneq \{ f: S → \K, f(s)$ ist beschränkt $\}$. +Dann wird $B(S)$ mit +\[ + \norm f _{B(S)} \coloneq \sup_{x ∈ S} |f(x)| < \infty , +\] +der $\sup$-Norm, zu einem Banachraum. +Dabei ist offensichtlich, dass $\norm\cdot_{B(S)}$ tatsächlich eine Norm ist, und wir werden in einer Übung zeigen, dass die induzierte Metrik tatsächlich vollständig ist. \begin{lemma-nn} \label{lemma-vollst-ubertragt-abgeschl-teilmengen} @@ -630,11 +713,13 @@ was bereits $\alpha = 0$ impliziert. Das ist ein Widerspruch. \end{proof} -\begin{beispiel-nn}[Räume stetiger Funktionen] - Sei $K ⊂ ℝ^n$ kompakt, also nach Heine-Borel abgeschlossen und beschränkt. +\subsection{Räume stetiger Funktionen} +\begin{beispiel-nn}[$C(K)$] + \index{$C(K)$} + Sei $K$ eine kompakte Teilmenge vom $ℝ^n$, also nach dem Satz von Heine-Borel abgeschlossen und beschränkt. Dann ist \[ - C(k) = \{ f: K → \K, f \text{ stetig} \} + C(K) = \{ f: K → \K, f \text{ stetig} \} \] ein normierter Raum mit \[ @@ -642,7 +727,7 @@ was bereits $\alpha = 0$ impliziert. Das ist ein Widerspruch. \] der Maximumsnorm. Dieses Maximum wird tatsächlich immer angenommen, da $K$ kompakt ist (Satz von Minimum und Maximum). - Insbesondere sind alle stetigen Funktionen auf $K$ beschränkt. Damit gilt offensichtlich $C(K) ⊂ B(K)$ und $\norm{f}_{C(K)} = \norm{f}_{B(K)}$ für alle $f ∈ C(K)$. + Insbesondere sind alle stetigen Funktionen auf $K$ beschränkt. Damit gilt offensichtlich $C(K) ⊂ B(K)$ und $\snorm{f}_{C(K)} = \snorm{f}_{B(K)}$ für alle $f ∈ C(K)$. Da jede stetige Funktion auf kompakten Teilmengen von metrischen Räumen auch gleichmäßig stetig ist, das heißt \[ ∀ \epsilon > 0 ∃ \delta > 0: \left( |t_1-t_2| < \delta \implies |f(t_1)-f(t_2)| < \epsilon \right) ∀ t_1,t_2 ∈ K @@ -654,7 +739,7 @@ was bereits $\alpha = 0$ impliziert. Das ist ein Widerspruch. \end{lemma} \begin{proof} Sei $(f_i)_{i ∈ ℕ}$ eine konvergente (in $(B(K),\norm\cdot_{B(K)})$) Folge in $C(K)$. - Dann existiert ein $f ∈ B(K)$ mit $f_i \xrightarrow[i → \infty ]{\norm{\cdot}_{B(K)}} f$. + Dann existiert ein $f ∈ B(K)$ mit $f_i \yrightarrow[i → \infty ]{\norm{\cdot}_{B(K)}} f$. Wir müssen zeigen, dass $f$ bereits stetig ist. Für beliebige $t₁, t_2 ∈ K$ gilt \[ @@ -678,7 +763,6 @@ Allerdings ist die Konvergenz in dieser Topologie impliziert keine Stetigkeit f \] Hier können Funktionen aber auch unbeschränkt sein. Also braucht $\sup |f|$ nicht mehr zu existieren. \end{beispiel-nn} - \begin{definition} Es sei $(K_m)_{m ∈ ℕ}$ eine \emph{Ausschöpfung} von $\Omega$ mit kompakten Mengen $K_= ⊂ \Omega$, das heißt, es gelte \[ @@ -693,7 +777,6 @@ Man nehme z.B. K_m = \{ x ∈ \Omega ⊂ ℝ^n: \norm{x} \le m, \operatorname{dist}(x,∂\Omega) \ge 1/m\}, \] wobei $\operatorname{dist}(x,∂\Omega) \coloneq \inf\{ \norm{x-y}: y ∈ ∂\Omega\}$ und $∂M = \cl \Omega \setminus \Omega$. - Dann ist $C(\Omega)$ mit der Metrik \[ d(f,0) = \sum_{m ∈ ℕ} 2^{-m} \frac{\norm{f}_{C(K_m)}}{1+\norm{f}_{C(K_m)}} @@ -702,11 +785,10 @@ ein Fréchetraum, also ein metrisierbarer linearer Raum nach \cref{satz-abzaehlb \[ \norm{f}_{C(K_m)} = 0 ∀ m ∈ ℕ \implies f = 0 ∈ C(\Omega). \] - Es gilt in diesem Raum \[ - d(f_i,f) \xrightarrow[i → \infty ]{} 0 \gdw - \norm{f_i-f}_{C(K_m)} \xrightarrow[i → \infty ]{} ∀m ∈ ℕ, + d(f_i,f) \yrightarrow[i → \infty ]{} 0 \gdw + \norm{f_i-f}_{C(K_m)} \yrightarrow[i → \infty ]{} ∀m ∈ ℕ, \] was ja gerade gleichmäßige Konvergenz auf jeder Kompakten Menge $K ⊂ \Omega$ bedeutet. Damit ist Stetigkeit der Folgenglieder $(f_i)_{i ∈ ℕ} ⊂ C(\Omega)$ impliziert Stetigkeit der Grenzfunktion $f ∈ C(\Omega)$, da Stetigkeit nur eine lokale Eigenschaft ist. @@ -842,13 +924,13 @@ Wir werden in der Übung sehen, dass $C(\Omega)$ mit dieser Metrik $d_{C(\Omega) Nach Definition sind sie aber auch Konvex, das heißt $(\D(\Omega),\T_\D)$ ist ein lokalkonvexer Hausdorff-Raum. \end{korollar} \begin{satz} - $ξ_m \xrightarrow[m → \infty ]{} 0 \gdw$ + $ξ_m \yrightarrow[m → \infty ]{} 0 \gdw$ \[ \begin{cases} (i), & \text{Es existiert $D$ offen mit $D ⊂⊂ \Omega$ und $ξ_m ∈ C_0^\infty (D)$ für alle $m ∈ ℕ$} \\ (ii), & \text{Für jedes $k ∈ ℕ$ gilt: - $\norm{ξ_m}_{C^k(\cl{\Omega})} \xrightarrow[m → \infty ]{} 0$} + $\norm{ξ_m}_{C^k(\cl{\Omega})} \yrightarrow[m → \infty ]{} 0$} \end{cases} \] \end{satz} @@ -864,8 +946,8 @@ Wir werden in der Übung sehen, dass $C(\Omega)$ mit dieser Metrik $d_{C(\Omega) \end{proof} \end{enumerate} \end{beispiel-nn} - \begin{beispiel-nn}[Lebesgue-integrierbare Funktionen] - Betrachten wir nun Lebesgue-integrierbare Funktionen. + \begin{beispiel-nn}[Lebesgue"=integrierbare Funktionen] + Betrachten wir nun Lebesgue"=integrierbare Funktionen. Bereits eingeführt wurden die Räume $\L^p(\Omega)$ und $L^p(\Omega)$, $0 < p < \infty $, wobei $\Omega ⊂ ℝ^n$ offen. Diese sind für $1 \le p < \infty $ normiert, und für $0 < p < 1$ quasi-normiert. Für $p = \infty $ setzen wir @@ -928,6 +1010,7 @@ Wir werden in der Übung sehen, dass $C(\Omega)$ mit dieser Metrik $d_{C(\Omega) ∫_\Omega f(t) \cdot \left[ \frac {∂} {∂t_i} h(t) \right] \dd t = ∫_{∂\Omega} f(t) h(t) \nu_i \dd S(t) - ∫_\Omega \left[ \frac {∂} {∂t_i} f(t) \right] h(t) \dd t, \] wobei $\nu = (\nu_1, …, \nu_n)^T$ die äußere Einheitsnormale ist. +\end{beispiel} \begin{bemerkung-nn} Ist $f$ oder $h ∈ \C_0^∞(\Omega)$, so verschwinden die Randterme. @@ -985,7 +1068,6 @@ Wir werden in der Übung sehen, dass $C(\Omega)$ mit dieser Metrik $d_{C(\Omega) \] ein Banachraum. \end{satz} -\end{beispiel} Seien $f_n → f ∈ L^p(\Omega), h ∈ C_0^\infty (\Omega)$. Dann @@ -1127,7 +1209,7 @@ Nun wollen wir so ein Konzept für Beschränktheit auch in allgemeinen metrische „⊂“: Sei $x ∈ X$. Setze $β_n = 1/n, n ∈ ℕ$. Dann gilt \[ - β_n x \xrightarrow[n → \infty ]{} 0, + β_n x \yrightarrow[n → \infty ]{} 0, \] also $β_n ∈ V$ für $n \ge n_0$. Damit haben wir aber $x ∈ n_0 V$. \end{proof} @@ -1136,12 +1218,17 @@ Nun wollen wir so ein Konzept für Beschränktheit auch in allgemeinen metrische Sei $(X,\T)$ ein topologischer linearer $T_2$-Raum und $K ⊂ X$ kompakt. Dann ist $K$ abgeschlossen und beschränkt. \end{satz} +\begin{bemerkung-nn} + Ohne die Hausdorff"=Eigenschaft gilt dies nicht. + Wenn wir beispiesweise $X = ℝ$ mit der Klumpentopologie betrachten, + ist jede Teilmenge von $ℝ$ kompakt, aber nur $\emptyset$ und $ℝ$ sind abgeschlossen. +\end{bemerkung-nn} + +Im Allgemeinen ist die Umkehrung falsch. \begin{definition-nn} + Sei $(X,)$ Falls auch die Umkehrung gilt, dann besitzt $X$ die \emph{Heine"=Borel"=Eigenschaft}. \end{definition-nn} -\begin{warnung-nn} - Im Allgemeinen ist die Umkehrung falsch. -\end{warnung-nn} \begin{proof} Nach einer Übungsaufgabe ist $K$ bereits abgeschlossen. Also müssen wir nur zeigen, dass $K$ auch beschränkt ist. @@ -1157,10 +1244,6 @@ Nun wollen wir so ein Konzept für Beschränktheit auch in allgemeinen metrische \] also folgt die Behauptung mit $\alpha = n_s$. $(*)$ gilt wegen der Kreisförmigkeit und $\left|\frac {n_i}{n_s}\right| \le 1$. \end{proof} -\begin{bemerkung-nn} - Ohne die Hausdorff-Eigenschaft gilt dies nicht. Gegenbeispiel: $X = ℝ$ mit der Klumpentopologie. - Dann ist jede Teilmenge von $ℝ$ kompakt, aber nur $\emptyset$ und $ℝ$ sind abgeschlossen. -\end{bemerkung-nn} \begin{definition} \begin{enumerate} @@ -1209,7 +1292,7 @@ Nun wollen wir so ein Konzept für Beschränktheit auch in allgemeinen metrische \item $A$ ist beschränkt und gleichgradig stetig, das heißt, \[ - \sup_{f ∈ A} |f(x)-f(y)|_{ℝ^m} \xrightarrow[|x-y|→ 0]{} 0. + \sup_{f ∈ A} |f(x)-f(y)|_{ℝ^m} \yrightarrow[|x-y|→ 0]{} 0. \] \end{enumerate} \end{satz} @@ -1222,11 +1305,11 @@ Nun wollen wir so ein Konzept für Beschränktheit auch in allgemeinen metrische \item $A$ ist im Mittel gleichgradig stetig, das heißt \[ - \sup_{f ∈ A} \norm{f(\cdot + h) - f}_{L^p(ℝ^n)} \xrightarrow[|h| → 0]{} 0. + \sup_{f ∈ A} \norm{f(\cdot + h) - f}_{L^p(ℝ^n)} \yrightarrow[|h| → 0]{} 0. \] \item \[ - \sup_{f ∈ A} \norm{f}_{L^p(ℝ^n \setminus B_R(0))} \xrightarrow[R → ∞]{} 0. + \sup_{f ∈ A} \norm{f}_{L^p(ℝ^n \setminus B_R(0))} \yrightarrow[R → ∞]{} 0. \] \end{enumerate} \end{satz} @@ -1242,7 +1325,7 @@ Im allgemeinen muss $T$ nicht notwendigerweise stetig sein: \begin{beispiel} Sei $X = \mathcal S = ℝ^∞$ und für $i ∈ ℕ$ $e_i$ der $i$-te Einheitsvektor. - Dann ist $\{e_i\}_{i ∈ ℕ}$ linear unabhängig, also gibt es nach \cref{01-basisergaenzungssatz} eine Basis $\{w_i\}_{i ∈ I}$ von $S$, die $\{e_i\}_{i ∈ ℕ}$ umfasst. + Dann ist $\{e_i\}_{i ∈ ℕ}$ linear unabhängig, also gibt es nach~\cref{satz:basisergaenzungssatz-1.1.3} eine Basis $\{w_i\}_{i ∈ I}$ von $S$, die $\{e_i\}_{i ∈ ℕ}$ umfasst. Definiere $T: X → ℝ$ linear durch die Bilder der Basisvektoren $T(e_i) \coloneq 1$ und $T(w) \coloneq 0$ für $w ∈ B \setminus \{ e_i\}_{i ∈ ℕ}$. Dann ist $T$ nicht stetig in $0$, denn $f (\lim_{i → ∞} e_i) = f(0) = 0 \ne 1 = \lim_{i → ∞} f(e_i)$. \end{beispiel} @@ -1295,7 +1378,7 @@ Hier gilt $M = \inf \{ c \ge 0:$ mit $C$ gilt (5) $\}$. $(5) \Rightarrow (1)$. Für $x, x_1 ∈ X$ gilt \[ - \norm{T(x) - T(x_1)} = \norm {T(x-x_1)} \le C \norm x-x_1 \xrightarrow[x → x_1]{} 0. + \norm{T(x) - T(x_1)} = \norm {T(x-x_1)} \le C \norm x-x_1 \yrightarrow[x → x_1]{} 0. \] Damit ist $T$ stetig in $x_1$. \end{proof} @@ -1382,10 +1465,10 @@ In topologischen linearen Räumen gilt dies jedoch nciht. \begin{proof} Es ist nur noch die Vollständigkeit zu zeigen. - Sei dazu $(T_n)_{n ∈ ℕ}$ eine Cauchy-Folge in $\L(X,Y)$. + Sei dazu $(T_n)_{n ∈ ℕ}$ eine Cauchy"=Folge in $\L(X,Y)$. Das heißt, für jedes $\epsilon > 0$ existiert ein $N_0$ mit $\norm {T_n - T_m} < \epsilon $ für $n, m > N_0$. Also mit \eqref{eq:61} $\norm {T_n x - T_mx} \le \norm {T_n - T_m} \norm x < \epsilon \norm x$ für alle $x ∈ X$ und $n,m > N_0$. - Insbesondere ist $(T_nx)_{n ∈ ℕ}$ eine Cauchy-Folge in $Y$. Da $Y$ vollständig ist, besitzt diese Folge einen Grenzwert $y_x ∈ Y$. + Insbesondere ist $(T_nx)_{n ∈ ℕ}$ eine Cauchy"=Folge in $Y$. Da $Y$ vollständig ist, besitzt diese Folge einen Grenzwert $y_x ∈ Y$. Wir definieren eine Abbildung \[ T: X → Y, x ↦ y_x. @@ -1400,7 +1483,7 @@ In topologischen linearen Räumen gilt dies jedoch nciht. \[ \norm{Tx} \xleftarrow[n → \infty ]{} \norm{T_nx } \le M \norm x, ∀ x ∈ X, \] - also die stetigkeit von $T$. + also die Stetigkeit von $T$. Jetzt zur Konvergenz: Für $\norm x \le$ 1 gilt \[ @@ -1451,7 +1534,7 @@ In topologischen linearen Räumen gilt dies jedoch nciht. \norm{ S_l - S_k} = \norm { \sum_{n=k+1}^l T^n} \le \sum_{n=k+1}^l \norm{ T^k} \le \sum_{n=k+1}^l \Theta ^n < \epsilon , \quad k, l \ge N_0. \] Damit ist $(S_k)_{k ∈ ℕ}$ eine Cauchy-Folge in $\L(X)$ und somit konvergent. - Sei $S$ der Grenzwert. Dann gilt für jedes $x ∈ X$ auch $S_k x \xrightarrow[k → \infty ]{\norm\cdot_{X}} Sx$, also damit ist für alle $x∈ X$ + Sei $S$ der Grenzwert. Dann gilt für jedes $x ∈ X$ auch $S_k x \yrightarrow[k → \infty ]{\norm\cdot_{X}} Sx$, also damit ist für alle $x∈ X$ \[ (\id - T) Sx = \lim_{k → \infty } (\id -T) S_k x = \lim_{k → \infty } \sum_{n=0}^k (T^n -T^{n-1})x = \lim_{k→\infty } x - T^{k+1}x = x. \] @@ -1552,5 +1635,5 @@ Damit sind in unendlich-dimensionalen normierten Räumen weder die Sphären noch %%% Local Variables: %%% mode: latex -%%% TeX-master: "funkana-ebook" +%%% TeX-master: "funkana" %%% End: diff --git a/ch05-hahn-banach.tex b/ch05-hahn-banach.tex index c058649..131ad0f 100644 --- a/ch05-hahn-banach.tex +++ b/ch05-hahn-banach.tex @@ -1,4 +1,4 @@ -\chapter{Der Satz von Hahn-Banach und seine Konsequenzen} +\chapter{Der Satz von Hahn-Banach \\ und seine Konsequenzen} \section{Fortsetzbarkeit linearer Funktionale} Wir fragen uns, ob sich Abbildungen so erweitern, dass gewisse Eigenschaften (wie z.B. Linearität oder Stetigkeit) erhalten bleiben. @@ -27,16 +27,16 @@ Wir fragen uns, ob sich Abbildungen so erweitern, dass gewisse Eigenschaften (wi Wir behaupten, dass $(A_0x_n)_{n ∈ ℕ}$ eine Cauchy-Folge in $Y$ ist. Dazu beachte, dass \[ - \norm{A_0 x_n - A_0 x_m}_{Y} \le \norm{A_0}_{\L(X_0,Y)} \norm{x_n-x_m} \xrightarrow[n,m → \infty ]{} 0. + \norm{A_0 x_n - A_0 x_m}_{Y} \le \norm{A_0}_{\L(X_0,Y)} \norm{x_n-x_m} \yrightarrow[n,m → \infty ]{} 0. \] Da $Y$ ein Banachraum ist, ist $(A_0x_n)_{n\ge1}$ konvergiert, etwa gegen $y$. Wir setzen $Ax \coloneq y$. Zunächst ist $A$ wohldefiniert, denn wenn $(z_n)_{n \ge 1}$ eine weitere Folge mit $\lim_{n → \infty } z_n = x$ ist, dann gilt - $z_n - x_n \xrightarrow[n→\infty ]{} 0$ und + $z_n - x_n \yrightarrow[n→\infty ]{} 0$ und \begin{align*} \norm{A_0 z_n - y} &\le \norm{A_0 z_n - A_0 x_n} + \norm{A_0 x_n - y} \\ & \le - \norm{A_0} \norm{z_n - x_n} + \norm{A_0 x_n - y} \xrightarrow[n→\infty ]{} 0. + \norm{A_0} \norm{z_n - x_n} + \norm{A_0 x_n - y} \yrightarrow[n→\infty ]{} 0. \end{align*} Offensichtlich ist $A$ eine Fortsetzung von $A_0$. Dass $A$ linear ist, ist ebenfalls klar. @@ -65,7 +65,7 @@ Wir fragen uns, ob sich Abbildungen so erweitern, dass gewisse Eigenschaften (wi \] und für $x ∈ X$ \[ - \norm{B_x - A_x} \le \norm{B_x - Bx_n} + \norm{Bx_n - Ax_n} + \norm{Ax_n - Ax} \xrightarrow[n→\infty ]{} 0, + \norm{B_x - A_x} \le \norm{B_x - Bx_n} + \norm{Bx_n - Ax_n} + \norm{Ax_n - Ax} \yrightarrow[n→\infty ]{} 0, \] da $A$ und $B$ stetig sind. Also $Bx = Ax$ für alle $x ∈ X$ und damit $B = A$. \end{proof} @@ -163,13 +163,12 @@ Ist $X_0$ nicht dicht in $X$, wird die Fortsetzung schwieriger. \begin{satz}[5.3.1] Sei $(X,\norm\cdot)$ ein normierter Raum über $ℝ$, $M ⊂ X$ abgeschlossen und konvex und $0 ∈ M$. - Dann existiert zu jedem $x_0 \not\in M$ ein $f ∈ X'$ mit \[ f(x_0) > 1 ∧ ∀ x ∈ M: f(x) \le 1. \] -\end{satz} Die Hyperebene $H = \{ x ∈ X: f(x) = 1 + \epsilon \}$ für $0 < \epsilon < f(x_0) < 1$ trennt also $x_0$ und $M$. +\end{satz} \begin{proof} Setze $2r \coloneq \inf_{y ∈ M} \norm{y - x_0}$ (positiv, da $M$ abgeschlossen). @@ -225,7 +224,7 @@ Die Hyperebene $H = \{ x ∈ X: f(x) = 1 + \epsilon \}$ für $0 < \epsilon < f( \] \end{proof} -\section{Einbettung von $X$ in seinen Bidualraum} +\section{Einbettung von \(X\) in seinen Bidualraum} Zunächst zur Motivation: Sei $X$ ein normierter linearer Raum. Dann existiert $X'$ und ist ein Banachraum. Aber dann existiert auch $X'' \coloneq (X')'$ und ist ebenfalls ein Banachraum. @@ -560,5 +559,5 @@ Wähle dafür $x = ±1$ an den Zwischenpunkten des Riemann"=Stieltjes"=Integrals %%% Local Variables: %%% mode: latex -%%% TeX-master: "funkana-ebook" +%%% TeX-master: "funkana" %%% End: \ No newline at end of file diff --git a/ch06-schwache-topologien.tex b/ch06-schwache-topologien.tex index 52a7f4d..ea30203 100644 --- a/ch06-schwache-topologien.tex +++ b/ch06-schwache-topologien.tex @@ -4,7 +4,7 @@ Wir wissen bereits, dass $\cl{B_1(0)}$ genau dann kompakt ist, wenn $\dim X < Wir suchen nun eine sinnvolle Topologie für $X$, die von der Normtopologie noch möglichst viele Eigenschaften mitnimmt, aber die Einheitskugel kompakt macht. -\section{Schwache und schwach$*$-Topologie} +\section{Schwache und schwach\(*\)-Topologie} Zu $x' ∈ X'$ fest und $ε > 0$ sei \begin{equation}\label{eq:11} @@ -80,10 +80,8 @@ Zu $x' ∈ X'$ fest und $ε > 0$ sei \begin{proof} Übung. \end{proof} -\begin{bemerkung-nn} - Wir schreiben für die schwache Konvergenz $x_n \xrightharpoonup[n → ∞]{} x_0$. - Im Beispiel zur Definition 5.4.5 haben wir gesehen, dass $e_i \xrightharpoonup[n→∞]{} 0$ in $\ell^2$. -\end{bemerkung-nn} +Wir schreiben für die schwache Konvergenz $x_n \yrightharpoonup[n → ∞]{} x_0$. +Im Beispiel zur Definition 5.4.5 haben wir gesehen, dass $e_i \yrightharpoonup[n→∞]{} 0$ in $\ell^2$. Nach Satz 1.6 sind alle $x' ∈ X'$ als Abbildungen \[ x': (X, \T_w) → \K @@ -125,11 +123,9 @@ Dann definiere \begin{proof} ähnlich wie bei der schwachen Topologie. \end{proof} - \begin{warnung-nn} Eine „schwach$*$-Topologie“ auf $X$ geht offenbar \emph{nicht}. \end{warnung-nn} - \begin{bemerkung} Die schwach$*$ ist eine weitere Abschwächung der schwachen Topologie. Betrachte dazu den kanonischen Isomorphismus @@ -144,7 +140,7 @@ Dann definiere \] Damit $U'(x,ε) ∈ \T_{w,X'}$, also \[ - \T_{w^*,X'} ⊂ \T_{w, X'} ⊂ \T_{s, X'} + \T_{w^*,X'} ⊂ \T_{w, X'} ⊂ \T_{s, X'}. \] \end{bemerkung} \begin{korollar-nn} @@ -153,19 +149,17 @@ Dann definiere \begin{proof} klar. \end{proof} - Für den Konvergenzbegriff gilt analog zu Satz 1.6 \begin{satz} Sei $(x'_n)_{n ∈ ℕ}$ eine Folge in $X'$. Dann konvergiert $(x_n')_{n ∈ ℕ}$ in $(X',\T_{w^*,X'})$ gegen $x_0' ∈ X'$ genau dann, wenn $\lim_{n → ∞} \lAngle x_n', x \rAngle = \lAngle x_0', x \rAngle$ für alle $x ∈ X$. - Wir schreiben dafür $x_n' \xrightharpoonup[n→∞]{*} x_0$. + Wir schreiben dafür $x_n' \yrightharpoonup[n→∞]{*} x_0$. \end{satz} \begin{proof} Übung. \end{proof} - \begin{bemerkung} \begin{enumerate} \item @@ -187,12 +181,12 @@ Für den Konvergenzbegriff gilt analog zu Satz 1.6 \begin{satz} \begin{enumerate} \item - Aus $x_k' \xrightharpoonup[n → ∞]{*} x'$ in $X'$ folgt + Aus $x_k' \yrightharpoonup[n → ∞]{*} x'$ in $X'$ folgt \[ \norm{x'}_{X'} \le \liminf_{k → ∞} \norm{x'_k}_{X'}. \] \item - Aus $x_k \xrightharpoonup[n → ∞]{} x$ in $X$ folgt + Aus $x_k \yrightharpoonup[n → ∞]{} x$ in $X$ folgt \[ \norm{x}_{X} ≤ \liminf_{k → ∞} \norm{x_k}_{X}. \] @@ -212,7 +206,7 @@ Für den Konvergenzbegriff gilt analog zu Satz 1.6 \] das heißt $\norm{x'}_{X'} ≤ M$, was die Behauptung impliziert. \item - Gelte $x_k \xrightharpoonup[k→∞]{} x$ + Gelte $x_k \yrightharpoonup[k→∞]{} x$ Wie gerade folgt für alle $x' ∈ X'$ \[ | \lAngle x', x \rAngle | ≤ @@ -230,7 +224,7 @@ Für den Konvergenzbegriff gilt analog zu Satz 1.6 \end{enumerate} \end{proof} -\section{Schwach- und schwach$*$-kompakte Einheitskugeln} +\section{Schwach- und schwach\(*\)-kompakte Einheitskugeln} \begin{satz} Sei $(X,\norm-)$ separabel. Dann ist die (stark) abgeschlossene Einheitskugel $\cl{B_1}(0) = \{ x' ∈ X': \norm{x'} ≤ 1\} ⊂ X'$ schwach$*$-folgenkompakt. @@ -238,7 +232,7 @@ Für den Konvergenzbegriff gilt analog zu Satz 1.6 \begin{proof} Sei $\{x_n\}_{n ∈ ℕ}$ eine dichte Teilmenge von $X$. Sei $(x'_{k ∈ ℕ})$ eine Folge in $X'$ mit $\norm{x'_k}_{X'} ≤ 1$ für alle $k ∈ ℕ$. - Wir konstruieren ein $x' ∈ \cl{B_1(0)}$ mit $x'_k \xrightharpoonup[k→∞]{*}$ (für eine Teilfolge). + Wir konstruieren ein $x' ∈ \cl{B_1(0)}$ mit $x'_k \yrightharpoonup[k→∞]{*}$ (für eine Teilfolge). Für $n ∈ ℕ$ ist $(\lAngle x_k', x_n \rAngle)_{k ∈ ℕ}$ eine beschräkte Folge in $\K$, denn $|\lAngle x'_k, x_n \rAngle | \le \norm{x_n} \cdot 1 \; (*)$. Durch das Diagonalverfahren finden wir eine Teilfolge $(x_{k_m})_{m ∈ ℕ}$, so dass für alle $n ∈ ℕ$ $\lim_{m → ∞} \lAngle x'_{k_m},x_n \rAngle$ existiert: @@ -288,7 +282,7 @@ Für den Konvergenzbegriff gilt analog zu Satz 1.6 Also $x' ∈ X'$ und $\norm{x'}_{X'} = \norm{x'}_{Y'} ≤ 1$. Also $x' ∈ \cl{B_1(0)} ⊂ X'$. - Bleibt noch zu zeigen: $x'_k \xrightharpoonup[k→∞]{*} x'$. + Bleibt noch zu zeigen: $x'_k \yrightharpoonup[k→∞]{*} x'$. Sei dazu $x ∈ X$ beliebig und $y ∈ Y$ mit $\norm{x-y}_{X} < ε$ (geht weil $\cl Y = X$). Dann \[ @@ -325,5 +319,5 @@ Für den Konvergenzbegriff gilt analog zu Satz 1.6 %%% Local Variables: %%% mode: latex -%%% TeX-master: "funkana-ebook" +%%% TeX-master: "funkana" %%% End: diff --git a/common.tex b/common.tex index fdf1455..60e3ae9 100644 --- a/common.tex +++ b/common.tex @@ -4,6 +4,8 @@ % \author{Prof. Dr. Maier-Paape} \date{WS 17/18} +\renewenvironment{bemerkung-nn}{\par}{\par} + \AtBeginDocument{ \newcommand\norm[1]{\left\|#1\right\|} \newcommand\snorm[1]{\|#1\|} @@ -39,16 +41,43 @@ \def\epsilon{\varepsilon} \def\Tnat{\ensuremath{\T_{\mathrm{nat}}}} \def\Tcof{\ensuremath{\T_{\mathrm{cof}}}} +\stackMath +\newcommand\yrightarrow[2][]{\mathrel{% + \setbox2=\hbox{\stackon{\scriptstyle#1}{\scriptstyle#2}}% + \stackunder[0pt]{% + \xrightarrow{\makebox[\dimexpr\wd2\relax]{$\scriptstyle#2$}}% + }{% + \scriptstyle#1\,% + }% +}} +\newcommand\yrightharpoonup[2][]{\mathrel{% + \setbox2=\hbox{\stackon{\scriptstyle#1}{\scriptstyle#2}}% + \stackunder[0pt]{% + \xrightharpoonup{\makebox[\dimexpr\wd2\relax]{$\scriptstyle#2$}}% + }{% + \scriptstyle#1\,% + }% +}} +\newcommand\yrightharpoondown[2][]{\mathrel{% + \setbox2=\hbox{\stackon{\scriptstyle#1}{\scriptstyle#2}}% + \stackunder[0pt]{% + \xrightharpoondown{\makebox[\dimexpr\wd2\relax]{$\scriptstyle#2$}}% + }{% + \scriptstyle#1\,% + }% +}} } \addbibresource{ref.bib} -\indexsetup{headers={\indexname}{\indexname}} +% \indexsetup{headers={\indexname}{\indexname}} +% \makeindex[columns=2,intoc=true] \makeindex \begin{document} \sloppy \maketitle \section*{Vorwort} +\spacedlowsmallcaps{HI das ist ein doofer text} \label{sec:vorwort} Dies ist eine Vorlesungsmitschrift, die nichts mit den Dozenten oder dem Lehrstuhl, der die Veranstaltung hält, zu tun hat. @@ -75,7 +104,13 @@ Es werden regelmäßig PDFs unter \url{https://hrnz.li/files/funkana/funkana.pdf \include{ch06-schwache-topologien} \nocite{*} -\printbibliography +\let\emph\em +\printbibliography[heading=bibintoc] \printindex \end{document} + +%%% Local Variables: +%%% mode: latex +%%% TeX-master: "funkana" +%%% End: diff --git a/funkana.tex b/funkana.tex index 744b415..372583e 100644 --- a/funkana.tex +++ b/funkana.tex @@ -1,8 +1,12 @@ \documentclass[ 12pt, - DIV=11, - twoside=false, + DIV=10, + BCOR=0mm, + twoside=true, chapterprefix=true, + headinclude=true, + footinclude=true, + usegeometry=true, headings=big]{skript} - +\usepackage[top=2cm,bottom=2cm]{geometry} \input{common.tex} \ No newline at end of file diff --git a/motivation.tex b/motivation.tex index e680534..b0ea5c1 100644 --- a/motivation.tex +++ b/motivation.tex @@ -23,7 +23,7 @@ Doch zunächst ein paar Probleme, für deren Lösung man die Funktionalanalysis \] ist, die durch \[ - Y = \left\{ u ∈ X: \int_0^\pi |u(x)|^2 dx = 1 \right\} + Y = \Big\{ u ∈ X: \int_0^\pi |u(x)|^2 dx = 1 \Big\} \] gegeben ist. Zwar ist $Y$ (in der $\L^2([0,\pi ])$-Metrik) beschränkt und abgeschlossen, jedoch nicht kompakt. @@ -75,5 +75,5 @@ Unser Ziel ist es zunächst, die beiden Strukturen zu erarbeiten. %%% Local Variables: %%% mode: latex -%%% TeX-master: "funkana-ebook" +%%% TeX-master: "funkana" %%% End: diff --git a/ref.bib b/ref.bib index 1a39582..90449a7 100644 --- a/ref.bib +++ b/ref.bib @@ -158,4 +158,13 @@ MRREVIEWER = {Jean Mawhin}, series={Springer-Lehrbuch Masterclass}, year={2010}, publisher={Springer Berlin Heidelberg} +} +@book{munkres2000topology, + title={Topology}, + author={Munkres, J.R.}, + isbn={9780131784499}, + lccn={99052942}, + series={Topology}, + year={2000}, + publisher={Prentice-Hall} } \ No newline at end of file diff --git a/skript.cls b/skript.cls index 0cdb41a..3fe51c9 100644 --- a/skript.cls +++ b/skript.cls @@ -27,10 +27,15 @@ \RequirePackage{mathtools} \RequirePackage{amsmath, amssymb} +\RequirePackage{stackengine} + +\RequirePackage{microtype} + \ifxetexorluatex \RequirePackage[babelshorthands]{polyglossia} \setdefaultlanguage{german} + \PolyglossiaSetup{german}{indentfirst=true} \RequirePackage{fontspec} \defaultfontfeatures{Mapping=tex-text} \RequirePackage{empheq} @@ -55,7 +60,7 @@ \def\rAngle{\rangle\rangle} \fi % fonts -\setkomafont{disposition}{\rmfamily} +\setkomafont{disposition}{\bfseries\rmfamily} \RequirePackage[obeyDraft, obeyFinal, @@ -65,40 +70,42 @@ prependcaption, textsize=small]{todonotes} -\setkomafont{chapter}{\Huge} -\addtokomafont{chapterprefix}{\raggedleft} -\renewcommand*{\chapterformat}{% - \hrulefill\enskip\mbox{\scalebox{3}{\thechapter\autodot}}\vspace{-0.6\baselineskip}} -\usepackage{etoolbox} -\newcommand\chaptertopline{\smash{\raisebox{-3.5\baselineskip}{\rule{\textwidth}{.4pt}}}} -\makeatletter -\preto{\@@makeschapterhead}{\chaptertopline} -\preto{\@@makechapterhead}{\ifunnumbered{chapter}{\chaptertopline}{}} -\makeatother +\setkomafont{chapter}{\fontsize{26}{24}\selectfont} +\let\raggedchapter\raggedleft \preto{\chapterheadendvskip}{\noindent\hrulefill\par} \RedeclareSectionCommand[ - beforeskip=12ex, - afterskip=2\baselineskip]{chapter} + beforeskip=15ex, + afterskip=25ex]{chapter} % fonts \RequirePackage{setspace} -\setstretch{1.1} -% \setlength\parskip{4pt} -% \setlength\parindent{0pt} +\setstretch{1.08} +\setlength\parskip{0pt} +\setlength\parindent{2em} \RequirePackage[amsmath, thmmarks, framed]{ntheorem} +\RequirePackage{silence} \RequirePackage[framemethod=tikz]{mdframed} +\WarningFilter{mdframed}{You got a bad break} -\RequirePackage[xindy]{imakeidx} +\RequirePackage{makeidx} +\RequirePackage[totoc]{idxlayout} \RequirePackage[unicode,colorlinks,bookmarksopen=true]{hyperref} +\makeatletter +\pdfstringdefDisableCommands{\let\(\fake@math} +\newcommand\fake@math{}% just for safety +\def\fake@math#1\){\detokenize{#1}} +\makeatother \RequirePackage[capitalise, nameinlink]{cleveref} +\RequirePackage{ifdraft} + \pagestyle{scrheadings} -\clearscrheadfoot -\ohead{\headmark} +% \clearscrheadfoot +% \ohead{\headmark} \cfoot{\pagemark} -\ifoot{\tiny Revision\gitVtags: \gitAbbrevHash{} (\gitAuthorDate)} -\automark{section} +\ifoptionfinal{}{\ofoot{\tiny Revision\gitVtags: \gitAbbrevHash{} (\gitAuthorDate)}} +% \automark[chapter]{section} \RequirePackage[ backend=biber, @@ -128,16 +135,18 @@ \end{tikzpicture}} \usepackage{enumitem} -\setenumerate{label=(\alph*)} +\setenumerate{label=(\alph*),leftmargin=2em} +\newlist{wenumerate}{enumerate}{1} +\setlist[wenumerate]{leftmargin=3em} \makeatletter \makeatletter \newtheoremstyle{mychange}% {\item[\hskip\labelsep \theorem@headerfont ##2\hskip 0.3em\ ##1\theorem@separator]}% - {\item[\hskip\labelsep \theorem@headerfont ##2\hskip 0.3em\ ##1\ (##3)\theorem@separator]} + {\item[\hskip\labelsep \theorem@headerfont ##2\hskip 0.3em\ ##1\ {\normalfont(##3)}\theorem@separator]} \newtheoremstyle{nonumbermychange}% {\item[\hskip\labelsep \theorem@headerfont ##1\theorem@separator]}% - {\item[\hskip\labelsep \theorem@headerfont ##1\ (##3)\theorem@separator]} + {\item[\hskip\labelsep \theorem@headerfont ##1\ {\normalfont(##3)}\theorem@separator]} \makeatother \DeclareDocumentCommand\newmdtheoremenv{s O{} m o m o }{% \IfBooleanTF{#1}{% @@ -157,20 +166,22 @@ \newcounter{defsatzusw} \def\newthm#1#2{ \newmdtheoremenv[ntheorem, + usetwoside=false, leftmargin=1em, - linewidth=6pt, + linewidth=0.5em, % linecolor=myurlcolor!20, linecolor=green!30, leftline=true, rightline=false, bottomline=false, topline=false, - innerleftmargin=1em, + innerleftmargin=0.5em, ]{#1}[defsatzusw]{#2} \newmdtheoremenv*[ntheorem, + usetwoside=false, leftmargin=1em, - linewidth=6pt, + linewidth=0.5em, % linecolor=myurlcolor!20, linecolor=green!30, leftline=true, rightline=false, bottomline=false, topline=false, - innerleftmargin=1em, + innerleftmargin=0.5em, ]{#1-nn}{#2} } \def\newdef#1#2{\newtheorem{#1}[defsatzusw]{#2}\newtheorem*{#1-nn}{#2}} @@ -194,15 +205,20 @@ \newdef{bemerkung}{Bemerkung} \newdef{bemerkungen}{Bemerkungen} \newdef{definition}{Definition} +\newdef{notation}{Notation} \newdef{warnung}{Warnung} -\newdef{frage}{Frage} +\newdef{achtung}{Achtung} \newdef{erinnerung}{Erinnerung} +\theoremindent=\parindent +\theoremheaderfont{\scshape} +\newdef{frage}{Frage} +\newdef{problem}{Problem} \newdef{beispiel}{Beispiel} \newdef{beispiele}{Beispiele} -\newdef{problem}{Problem} \theoremstyle{nonumberplain} \theoremheaderfont{\itshape} +\theoremindent=0pt \theorembodyfont{\normalfont} \theoremseparator{.} % \theoremsymbol{\scalebox{0.8}{\ensuremath{\blacksquare}}} @@ -214,5 +230,17 @@ \renewcommand{\thesection}{\arabic{section}} \renewcommand\sectionformat{\S \thesection\autodot \hspace{1ex}} +\renewcommand\subsectionformat{\S \thesubsection\autodot \hspace{1ex}} +%\usepackage{textcase} +\ifxetex + \DeclareRobustCommand{\spacedlowsmallcaps}[1]{{\scshape\MakeLowercase{#1}}}% +\else + \DeclareRobustCommand{\spacedlowsmallcaps}[1]{\textls[80]{\scshape\MakeLowercase{#1}}}% +\fi +\setkomafont{section}{\Large\normalfont\spacedlowsmallcaps} +\setkomafont{subsection}{\large\normalfont\spacedlowsmallcaps} + +\let\emph\relax +\DeclareTextFontCommand{\emph}{\bfseries} \endinput \ No newline at end of file -- cgit v1.2.3-24-g4f1b